<<

LIVE Test – 5 2nd May 2020

Q1. With reference to the Economic Survey 2019-20, consider the following statements: 1. As per World Bank’s Data on Entrepreneurship, ranks third in number of new firms created. 2. There is a direct relationship between registration of new firms and Gross Domestic District Product (GDDP). Which of the statements given above is/are correct? a. 1 only b. 2 only c. Both 1 and 2 d. Neither 1 nor 2

Answer: c

Explanation:

The Economic Survey 2019-20 states that as per World Bank’s Data on Entrepreneurship, it is seen that India ranks third in number of new firms created. Key highlights of survey: o Entrepreneurship as a strategy to fuel productivity growth and wealth creation. o The Survey stated that as per World Bank’s Data on Entrepreneurship, it is seen that India ranks third in number of new firms created. o The same data shows that new firm creation has gone up dramatically in India since 2014. o 12.2 % cumulative annual growth rate of new firms in the formal sector during 2014- 18, compared to 3.8 % during 2006-2014. o About 1.24 lakh new firms created in 2018, an increase of about 80 % from about 70,000 in 2014. • New firm creation in services is significantly higher than that in manufacturing, infrastructure or agriculture. • A 10 percent increase in registration of new firms in a district yields a 1.8 % increase in Gross Domestic District Product (GDDP). 1

• Literacy and education in a district foster local entrepreneurship significantly: o Impact is most pronounced when literacy is above 70 per cent. o New firm formation is the lowest in eastern India with lowest literacy rate (59.6 % as per 2011 Census). • The Survey notes that the entrepreneurial activity in the Manufacturing sector is highest in the regions of , Meghalaya, Puducherry, Punjab and Rajasthan. Hence both statements are correct. Source: https://vajiramias.com/current-affairs/entrepreneurship-and-wealth-creation-at- the-grassroots/5e34fb271d5def055b0e8c22/ Q2. 'Beijing Declaration and Platform for Action', often seen in the news, is a. a strategy to tackle the regional terrorism, an outcome of a meeting of the Shanghai Cooperation Organization b. a plan of action for sustainable economic growth in the Asia-Pacific Region, an outcome of the deliberations of the Asia-Pacific Economic Forum c. an agenda for women's empowerment, an outcome of a World Conference convened by the United Nations d. a strategy to combat wildlife trafficking, a declaration of the East Asia Summit Answer: c Explanation: To mark of 25 years of the adoption of Beijing Platform for Action, Ministry of Women & Child Development (MWCD), recently organized a National Consultation on the Review of Beijing+25. The 1995 Fourth World Conference on Women, held in Beijing, was one of the largest ever gatherings of the United Nations, and a critical turning point in the world’s focus on gender equality and the empowerment of women. The Beijing Declaration was a resolution adopted by the UN at the end of the Fourth World Conference on Women. The resolution adopted to promulgate a set of principles concerning the equality of men and women. 2020 marks the 25th anniversary of the Fourth World Conference on Women and adoption of the Beijing Declaration and Platform for Action (1995), (Beijing + 25). Hence, option (c) is the correct answer. Source: https://vajiramias.com/current-affairs/beijing-declaration-and-platform-for-action- 1995/5e34f9c01d5def0558bfa515/ Q3. Surajkund is an ancient reservoir of the 10th century located in: a. Haryana b. Uttar Pradesh

2

c. Bihar d. Punjab Answer: a Explanation: The President of India, recently inaugurated the 34th Surajkund International Crafts Mela. This year the partner country in the fair is Uzbekistan and Himachal Pradesh is the theme-state. Surajkund is an ancient reservoir of the 10th century located on Southern Ridge of Aravalli range in Faridabad city of Haryana. Surajkund (literal meaning is 'Lake of the Sun') is an artificial Kund ('Kund' means "lake" or reservoir). It is said to have been built by the king Surajpal of Tomar dynasty in the 10th century. Surajkund is known for its annual fair "Surajkund International Craft Mela", which is being organized since 1987 to showcase the richness and diversity of handicrafts, handlooms and cultural heritage of India. Hence, option (a) is the correct answer. Source: https://vajiramias.com/current-affairs/surajkund-international-crafts- mela/5e34fa4b1d5def0558bfa525/ Q4. With reference to the NIRVIK scheme, consider the following statements: 1. It provides for higher credit disbursement for both import and export. 2. The Scheme is being prepared by the Ministry of Finance. Which of the statements given above is/are correct? a. 1 only b. 2 only c. Both 1 and 2 d. Neither 1 nor 2

Answer: d

Explanation:

Statement 1 is incorrect: The Finance Minister recently announced the NIRVIK (Niryat Rin Vikas Yojana) scheme in the Union Budget 2020-21 in Parliament. To achieve higher export credit disbursement, a new scheme NIRVIK is being launched which provides for high insurance cover, reduction in premium for small exporters and simplified procedures for claim, settlement.

3

Statement 2 is incorrect: The Scheme is being prepared by the Commerce and Industry Ministry. The commerce and industry ministry is working on the scheme also called the Export Credit Insurance Scheme (ECIS) under which the insurance guaranteed could cover up to 90% of the principal and interest. At present, the Export Credit Guarantee Corporation provides credit guarantee of up to 60% loss. Source: https://vajiramias.com/current-affairs/nirvik-scheme/5e364b481d5def05592c4a4a/ Q5. With reference to the ‘Vivad Se Vishwas’ Scheme, consider the following statements: 1. It aims at reducing litigations in the direct as well as indirect tax payments. 2. It is specially launched keeping in mind fall in revenue income due to Covid-19 lockdown.. Which of the statements given above is/are correct? a. 1 only b. 2 only c. Both 1 and 2 d. Neither 1 nor 2

Answer: d

Explanation:

Union Finance Minister has recently proposed wide-ranging facilitation measures in Direct Tax Regime while presenting the union budget. The Union Budget has proposed ‘Vivad Se Vishwas’ Scheme (No dispute but trust) which aims at reducing litigations in the direct taxes payments. It was launched before the Covid-19 lockdown began. Under the proposed scheme, a taxpayer would be required to pay only the amount of the disputed taxes and will get complete waiver of interest and penalty provided he pays by 31st March, 2020. Those who avail this scheme after 31st March, 2020 will have to pay some additional amount. The scheme will remain open till 30th June, 2020. To impart greater efficiency to the assessment process, a new faceless assessment scheme has already been introduced. Hence both statements are incorrect. Source: https://vajiramias.com/current-affairs/direct-tax- regime/5e3647f41d5def056166a182/ Q6. With reference to the Archimedes Principle, consider the following statements:

4

1. It refers to the law of buoyancy. 2. If the buoyant force is more than the weight, the object sinks; if it is less, the object rises. Which of the statements given above is/are correct? a. 1 only b. 2 only c. Both 1 and 2 d. Neither 1 nor 2

Answer: a

Explanation:

Statement 1 is correct: In physics, the Archimedes Principle refers to the law of buoyancy (the ability or tendency of something to float in water or other fluids). According to the principle, when an object is completely or partially submerged in a fluid, whether gas or liquid, it is acted upon by an upward force (buoyancy) equal to the weight of the fluid it has displaced. The force acting downward on the object is the weight of the object. The upward force is the one given by the Archimedes Principle. The difference between the two forces is the net force acting on the object. Statement 2 is incorrect: If the buoyant force is more than the weight, the object rises; if it is less, the object sinks. If the net force is zero, the object remains in place, and neither rises nor sinks. In the case of the elephant rescued in Jharkhand, the forest officials pumped water into the well so that the elephant could float to the surface. Source: https://vajiramias.com/current-affairs/archimedes- principle/5e3797841d5def055b0f33b3/ Q7. With reference to the Classical Swine Fever (CSF), consider the following statements: 1. It is one of the most important diseases of pigs causing high mortality. 2. Till date, there has been no vaccine developed to to Which of the statements given above is/are correct? a. 1 only

5

b. 2 only c. Both 1 and 2 d. Neither 1 nor 2 Answer: a Explanation: Indian Council of Agricultural Research (ICAR) recently released the Live attenuated Classical Swine Fever Vaccine (IVRI-CSF-BS) Technology developed by ICAR -Indian Veterinary Research Institute (IVRI), Izatnagar. Classical Swine Fever (CSF) is one of the most important diseases of pigs causing high mortality with an annual loss of approx. Rs.4.299 billion. A laminated CSF vaccine (Weybridge strain, UK) is being used in India since 1964 for controlling the disease. The vaccine is produced by sacrificing large numbers of rabbits for each batch. In order to do away sacrificing of rabbits and increase productivity, IVRI has developed a new CSF Cell Culture Vaccine by attenuating an indigenous virulent CSF virus in cell culture. The vaccine is safe, potent, does not revert to virulence and provide protective immunity from day 14 of the vaccination until 24 months studied so far. The vaccine has been tested on around 500 pigs at multiple locations. The new vaccine will be part of the Government’s One Health Initiative. Hence only statement 1 is correct. Source: https://vajiramias.com/current-affairs/live-attenuated-classical-swine-fever-vaccine- ivri-csf-bs/5e38feed1d5def05592d0628/ Q8. Sophisticated Analytical & Technical Help Institutes (SATHI) is an initiative of: a. Union Ministry of Science & Technology b. Union Ministry of Social Justice and Empowerment c. Union Ministry of Human Resource Development d. Union Ministry of Information and Broadcasting Answer: a Explanation: The Department of Science & Technology has recently launched a unique scheme called Sophisticated Analytical & Technical Help Institutes (SATHI) to address the need for building shared, professionally managed and strong Science and Technology infrastructure in the country. These Centres are expected to house major analytical instruments to provide common services of high-end analytical testing, thus avoiding duplication and reduced dependency on foreign sources. These would be operated with a transparent, open access policy. DST has

6

already set up three such centres in the country, one each at IIT Kharagpur, IIT Delhi and BHU at a total cost of Rs 375 Cores. It is planned to set up five SATHI Centres every year for the next four years. SATHI will address the problems of accessibility, maintenance, redundancy and duplication of expensive equipment in our Institutions, while reaching out to the less endowed organizations in need, e.g., industry, MSMEs, startups and State Universities. Hence, option (a) is the correct answer. Source: https://vajiramias.com/current-affairs/sophisticated-analytical-technical-help- institutes-sathi/5e38ff931d5def05592d0660/ Q9. Addu Atoll which is also known as Seenu Atoll, recently seen in the news, is located in: a. United Arab Emirates (UAE) b. Maldives c. Sri Lanka d. Seychelles Answer: b Explanation: India and Maldives have recently signed five MoUs for establishing the Addu zone in five islands of Addu atoll at a cost of 2.49 million dollars. A 6th MoU to set up a bottled water plant in Hoarafushi was also signed. Addu Atoll, also known as Seenu Atoll, is the southernmost atoll of the Maldives. Administratively, Addu Atoll is the location of Addu City, one of the two cities of the Maldives. Addu City consists of the inhabited areas of Addu Atoll, namely the natural islands of Hulhudhoo, Meedhoo, Maradhoo, Feydhoo, and Hithadhoo. Addu Atoll, together with Fuvahmulah, extend the Maldives into the Southern Hemisphere. Hence, option (b) is the correct answer. Source: https://vajiramias.com/current-affairs/addu-tourism- zone/5e3900641d5def055ad4b895/ Q10. With reference to the Gram Nyayalayas Act, 2008, consider the following statements: 1. It is an Act of Parliament enacted for establishment of Gram Nyayalayas or village courts for speedy and easy access to the justice system in the rural areas of India. 2. The Gram Nyayalayas have only civil jurisdiction over the offences. Which of the statements given above is/are correct? a. 1 only b. 2 only

7

c. Both 1 and 2 d. Neither 1 nor 2 Answer: a Explanation: Statement 1 is correct: Gram Nyayalayas Act, 2008 is an Act of Parliament enacted for establishment of Gram Nyayalayas or village courts for speedy and easy access to the justice system in the rural areas of India. Sections 5 and 6 of the 2008 Act provide that state government in consultation with the high court will appoint a ‘Nyayadhikari’ for each ‘Gram Nyayalaya’, who will be a person eligible to be appointed as a Judicial Magistrate of the First Class. Statement 2 is incorrect: The Gram Nyayalayas have both civil and criminal jurisdiction over the offences and nature of suits specified in the First, Second and Third Schedule of the Act. Source: https://vajiramias.com/current-affairs/gram-nyayalayas-act- 2008/5e3901311d5def05592d068d/ Q11. With reference to the Commonwealth, consider the following statements: 1. It is a voluntary association of 54 independent states, all of which were former territories of the British Empire. 2. The current Commonwealth of Nations was constituted by the London Declaration in 1949. Which of the statements given above is/are correct? a. 1 only b. 2 only c. Both 1 and 2 d. Neither 1 nor 2 Answer: b Explanation: The Commonwealth is a voluntary association of 54 independent and equal sovereign states, nearly all of them former territories of the British Empire. Headquarters is in London, United Kingdom. The current Commonwealth of Nations was formally constituted by the London Declaration in 1949. 54 countries are members of the Commonwealth with Maldives becoming the 54th member of Commonwealth family. All members have an equal say – regardless of size 8

or economic stature. This ensures even the smallest member countries have a voice in shaping the Commonwealth. Every two years, members meet to discuss issues affecting the Commonwealth and the wider world at the Commonwealth Heads of Government Meeting (CHOGM). Although the majority of the members are former British colonies, there are a few countries which were not. Mozambique and Rwanda are such prominent examples. Hence only statement 2 is correct. Source: https://vajiramias.com/current- affairs/commonwealth/5e3901671d5def05592d0692/ Q12. With reference to the Anganwadis, consider the following statements: 1. They are set up under the Integrated Child Development Services (ICDS) by the Ministry of Health and Family Welfare to provide a package of six services. 2. The aim of the scheme is to reduce infant mortality and child malnutrition. Which of the statements given above is/are correct? a. 1 only b. 2 only c. Both 1 and 2 d. Neither 1 nor 2

Answer: b

Explanation:

Anganwadis or day-care centres are set up under the Integrated Child Development Services (ICDS) by the Women and Child Development Ministry to provide a package of six services. The services include supplementary nutrition; pre-school non-formal education; immunisation, nutrition and health education; as well as referral services. The aim of the scheme is to reduce infant mortality and child malnutrition. Beneficiaries include children in the age group of six months to six years, and pregnant women and lactating mothers. Hence only statement 2 is correct. Value Addition Integrated Child Development Scheme is a centrally sponsored scheme implemented by state governments and union territories. It aims at providing supplementary nutrition, immunization and pre-school education to the children.

9

Source: https://vajiramias.com/current-affairs/anganwadi/5e3a3b651d5def055ad4f4fc/ Q13. With reference to the Purified Terephthalic Acid (PTA), consider the following statements: 1. It is an important raw material used to make various products, including polyester fabrics. 2. It makes up for around 70-80% of a polyester product. Which of the statements given above is/are correct? a. 1 only b. 2 only c. Both 1 and 2 d. Neither 1 nor 2

Answer: c

Explanation:

Purified Terephthalic Acid (PTA) is a crucial raw material used to make various products, including polyester fabrics. PTA makes up for around 70-80% of a polyester product. This includes products like polyester staple fibre and spun yarn. Some sportswear, swimsuits, dresses, trousers, curtains, sofa covers, jackets, car seat covers and bed sheets have a certain proportion of polyester in them. Recently the Government of India decided to abolish in “public interest” an anti-dumping duty that was levied on imports of Purified Terephthalic Acid (PTA). Hence both statements are correct. Source: https://vajiramias.com/current-affairs/purified-terephthalic-acid- pta/5e3a3ac41d5def055ad4f4f2/ Q14. With reference to the Global Misery Index (GMI), consider the following statements: 1. It is annually released by Organisation for Economic Co-operation and Development (OECD). 2. It is based on the three parameters namely unemployment rate, inflation rate and lending rate. Which of the statements given above is/are correct? a. 1 only b. 2 only

10

c. Both 1 and 2 d. Neither 1 nor 2

Answer: b

Explanation:

Statement 1 is incorrect: American economist Steve Hanke of Johns Hopkins University, an applied economist has popularised the Global Misery Index (GMI) concept. He has recently ranked India a measly 44 out of 95 countries on GMI. Statement 2 is correct: The benchmark index measures people’s “misery score” instead of the conventional gross domestic product (GDP). It is based on the three parameters namely unemployment rate, inflation rate and lending rate. To calculate the misery index, the annualised growth of GDP is subtracted from the sum total of these three rates. That gives a score which really defines how miserable people living in particular geography are. Source: https://vajiramias.com/current-affairs/global-misery-index- gmi/5e3b81c71d5def055b1007e2/ Q15. With reference to flame-throated bulbul, consider the following statements: 1. It is found only in the forests of the Eastern in southern India. 2. IUCN Red List Status is vulnerable. Which of the statements given above is/are correct? a. 1 only b. 2 only c. Both 1 and 2 d. Neither 1 nor 2

Answer: d

Explanation:

Flame-throated bulbul is a member of the bulbul family of passerine birds. They are olive backed with yellow undersides and a triangular orange-red throat that stands out against the contrasting blackhead. It is found in the from southern and 11

southwards. IUCN Red List Status is Least Concern. It was the official mascot for the 36th National Games held in Goa. It is the State bird of Goa. Hence both statements are incorrect. Source: https://vajiramias.com/current-affairs/rubigula/5e3b812d1d5def0558c10fa0/ Q16. The Union Cabinet has recently given its 'in-principle' approval for setting up a Major port at: a. Vadhavan, Maharashtra b. Mangaluru, c. Puri, d. Kanyakumari, Answer: a Explanation: The Union Cabinet has recently given its 'in-principle' approval for setting up a Major Port at Vadhavan near Dahanu in Maharashtra. Vadhavan port will be developed on the "landlord model". A Special Purpose Vehicle (SPV) will be formed with Jawaharlal Nehru Port Trust (JNPT) as the lead partner with equity participation equal to or more than 50% to implement the project. The SPV will develop the port infrastructure including reclamation, construction of a breakwater, besides establishing connectivity to the hinterland. All the business activities would be undertaken under PPP mode by private developers. The total cost of the project is likely to be Rs.65, 544 crores. The position of JN Port, the biggest container port in India is 28th in the world with a traffic of 5.1 million TEUs (Twenty-Foot Equivalent Units). Even after the completion of 4th terminal at JN Port with a capacity increase upto10 million TEUs by 2023, it will stand as the 17thlargest container port in the world. With the development of Vadhavan port, India will break into the countries with top 10 container ports in the world. Hence, option (a) is the correct answer. Source: https://vajiramias.com/current-affairs/vadhavan- port/5e3b82571d5def056167eade/ Q17. Lucknow Declaration, recently seen in the news, is related to: a. BRICS b. First India-Africa Defence Ministers’ Conclave (IADMC 2020) c. SAARC summit 2020 d. None of the above

12

Answer: b Explanation: The first India-Africa Defence Ministers’ Conclave (IADMC 2020) held in Lucknow recently, coinciding with DefExpo-2020, adopted the Lucknow Declaration. Besides traditional partners in Eastern and Southern Africa, Western African states have also sought to deepen defence ties with India including training for its officers and joint defence exercises. The leaders recognised the importance of peace and security for both India and the African countries including "Silence the Guns; Creating Conducive Conditions for African Development” as the African Union’s theme of the year. They welcomed the AU vision for peace and security in Africa that coincides with India’s vision of SAGAR (Security And Growth for All in the Region). Defence Ministers also called for deeper cooperation in the domain of defence industries including through investments, a joint venture in defence equipment software, digital defence, research and development on mutually beneficial terms. India has also made available defence equipment and supply to African countries through its robust India-Africa development partnership, including through grant assistance and Lines of Credit. This effort will be considerably enhanced. Hence, option (b) is the correct answer. Source: https://vajiramias.com/current-affairs/lucknow- declaration/5e3cefaa1d5def055ad5887a/ Q18. Brihadeshwara Temple is located in South Bank of which of the following river? a. b. Narmada c. Pennar d. Answer: a Explanation: The Kumbhabishekam (consecration) ceremony at the Sri Brahadeeswarar Temple was recently held after 23 years. Brihadeshwara Temple is also called as Rajarajesvaram or Peruvudaiyar Kovil temple. It is a dedicated to . The Kumbam (the apex or the bulbous structure on the top) of the temple is carved out of a single rock. It is located in South Bank of Kaveri river in Thanjavur, Tamil Nadu. Peruvudaiyaar Kovil is an example of Tamil architecture from the Chola period. It was built by Tamil King Raja Raja Chola I and completed in 1010 AD. The temple is part of the UNESCO World Heritage Site known as the "Great Living Chola Temples", along with Gangaikonda Cholapuram and Airavatesvara temple. Hence, option (a) is the correct answer.

13

Source: https://vajiramias.com/current-affairs/brihadeeswarar- temple/5e3cf0ee1d5def055ad588eb/ Q19. Namaste Orchha is a three-day multi-cultural festival, recently seen in the news, to promote tourism in: a. Chattisgarh b. Uttar Pradesh c. d. Haryana Answer: b Explanation: Madhya Pradesh government recently organized a three-day multi-cultural festival to promote tourism in the heritage city of Orchha, Madhya Pradesh. It celebrated the cultural diversity, natural beauty and the architectural heritage of the town by showcasing music, dance, heritage walks, local cuisine, arts and handicrafts. The first day of the festival held at Jahangir Mahal, built in 17th Century by the then ruler Vir Singh Deo in honour of the Mughal emperor Jahangir. Noted Hindustani classical vocal singer Shubha Mudgal performed alongside a giant African Baobab, a 500-year-old tree planted by Maharaja Vir Singh Deo. Orchha (or Urchha) is a town in Niwari district of Madhya Pradesh. Orchha lies on the . Orchha was founded in the 16th century AD by the Bundela Rajput chief, Pratap Singh, who became the first King of Orchha and also built the Fort of Orchha. Hence, option (b) is the correct answer. Source: https://vajiramias.com/current-affairs/namaste- orchha/5e3cef211d5def055ad58855/ Q20. With reference to the National Means-cum-Merit Scholarship Scheme (NMMSS), consider the following statements: 1. It is a centrally sponsored scheme. 2. The objective is to award scholarships to meritorious students of economically weaker sections to arrest their dropping out of school. Which of the statements given above is/are correct? a. 1 only b. 2 only c. Both 1 and 2

14

d. Neither 1 nor 2

Answer: b

Explanation:

National Means-cum-Merit Scholarship Scheme (NMMSS) is a Central Sector Scheme. Implemented since 2008. Objective is to award scholarships to meritorious students of economically weaker sections to arrest their drop out at class VIII and encourage them to continue the study at secondary stage. Under the Scheme one lakh fresh scholarships @ of Rs.12000/- per annum per student are awarded to selected students of class IX every year and their continuation/renewal in classes X to XII for study in a State Government, Government- aided and Local body schools. The scheme, NMMSS is boarded on National Scholarship Portal (NSP) since 2015-16. The NSP has been developed by Department of Electronics and Information Technology (DeitY) for streamlining and fast tracking the release of Scholarships across Ministries / Departments with efficiency, transparency and reliability. Hence only statement 2 is correct. Source: https://vajiramias.com/current-affairs/national-means-cum-merit-scholarship- scheme-nmmss/5e3cf0601d5def055ad588b9/ Q21. With reference to the 15th Finance Commission, consider the following statements: 1. The Commission has reduced the share of tax revenues that the Centre shares with the states from 42% to 32%. 2. The Commission intends to set up an expert group to initiate a non-lapsable fund for defence expenditure. Which of the statements given above is/are correct? a. 1 only b. 2 only c. Both 1 and 2 d. Neither 1 nor 2

Answer: b

Explanation:

The report of the 15th Finance Commission, along with an Action Taken Report, was tabled in Parliament.

15

• The Commission has reduced the vertical devolution — the share of tax revenues that the Centre shares with the states — from 42% to 41%. • The Commission intends to set up an expert group to initiate a non-lapsable fund for defence expenditure. It may do so by creating a separate fund from the gross tax revenue before computing the divisible pool — which means that states would get a smaller share of the taxes. • It has considered the 2011 population along with forest cover, income distance, tax effort, area of the state, and “demographic performance” to arrive at the states’ share in the divisible pool of taxes. • The use of 2011 population figures has resulted in states with larger populations like Uttar Pradesh and Bihar getting larger shares, while smaller states with lower fertility rates have lost out. Shares of the southern states, except Tamil Nadu, have fallen — with Karnataka losing the most. • In order to reward population control efforts by states, the Commission developed a criterion for demographic effort — which is essentially the ratio of the state’s population in 1971 to its fertility rate in 2011 — with a weight of 12.5%. But its impact is not entirely clear. • The Income distance criterion: Income distance is calculated as the difference between the per capita gross state domestic product (GSDP) of the state from that of the state with the highest per capita GSDP, with states with less income getting a higher share in order to allow them to provide services comparable to those provided by the richer ones. View of centre on recommendations:

• The Centre has recently rejected the 15th Finance Commission’s recommendation to give special grants worth ₹6,764 crore to States in 2020-21 to ensure that they do not receive less than the previous year’s allocation. • Major recommendations accepted by the Centre include the 41% share for States out of the divisible pool of tax collections, the suggested grants-in-aid and post-devolution revenue deficit grants of ₹74,340 crore for 14 States. It also accepted recommendations for grants to local bodies, disaster-related grants and sectoral grants. Hence only statement 2 is correct. Source: https://vajiramias.com/current-affairs/15th-finance- commission/5e3ceec21d5def055b104726/ Q22. What is Sharang, recently seen in the news? a. Artillery gun b. Submarine c. Naval version of pinaka rocket

16

d. None of the above Answer: a Explanation: The Ordnance Factory Board (OFB) recently handed over Sharang, the first 130mm M-46 artillery gun upgraded to 155mm to the Indian Army. Sharang is the 130mm artillery gun ‘up- gunned’ to 155mm, 45 calibre up-gunning based on the Army’s tender. The gun’s range has now gone from 27 km to over 36 km with the upgrade. It also has more explosive capability and hence and more damage potential. In all, OFB will upgrade 300 130mm guns to 155mm and the contract will be completed in four years. The Army inducted its first modern artillery guns system in November 2018. These include M-777 Ultra Light Howitzers (ULH) from the U.S. and K9 Vajra-T self-propelled artillery guns. The Army has the older, battle-proven Bofors 155mm guns in service. The 155mm Dhanush towed gun system, developed based on the Bofors guns, is under induction. Hence, option (a) is the correct answer. Source: https://vajiramias.com/current-affairs/sharang/5e3e21761d5def2725495675/ Q23. With reference to the International Gandhi Peace Prize, consider the following statements: 1. It is awarded biennially by the Government of India. 2. The award carries ₹ 1 Crore in cash and open to all persons regardless of nationality, race, creed or gender. Which of the statements given above is/are correct? a. 1 only b. 2 only c. Both 1 and 2 d. Neither 1 nor 2

Answer: b

Explanation:

The International Gandhi Peace Prize is awarded annually by the Government of India. It was instituted in 1995 to honour individuals and institutions for their contributions towards social, economic and political transformation through non-violence and other Gandhian methods. The award carries an amount of Rs 1Crore, a Citation in a scroll, a plaque as well as an exquisite traditional handicraft/handloom item. It is open to all persons regardless of nationality, race,

17

creed or gender. The award was instituted in 1995 during the commemoration of the 125th birth anniversary of Mahatma Gandhi and is to be given annually. Hence only statement 2 is correct. Source: https://vajiramias.com/current-affairs/international-gandhi-awards-for- leprosy/5e3cef6c1d5def055b104765/ Q24. With reference to the Thwaites Glacier, consider the following statements: 1. It is an Antarctic glacier flowing into Pine Island Bay, part of the Amundsen Sea. 2. It has been described as part of the "weak underbelly" of the West Antarctic Ice Sheet. Which of the statements given above is/are correct? a. 1 only b. 2 only c. Both 1 and 2 d. Neither 1 nor 2

Answer: c

Explanation:

A new study has pinned the cause of the melting of Thwaites Glacier to the presence of warm water at a vital point beneath the glacier. Thwaites Glacier is an Antarctic glacier flowing into Pine Island Bay, part of the Amundsen Sea. It is unusually broad and fast glacier with its surface speeds exceeding 2 km/yr near its grounding line. The grounding line is the place below a glacier at which the ice transitions between resting fully on bedrock and floating on the ocean as an ice shelf. The location of the line is a pointer to the rate of retreat of a glacier. Along with Pine Island Glacier, Thwaites Glacier has been described as part of the "weak underbelly" of the West Antarctic Ice Sheet. It is predicted that it will gradually melt, leading to an irreversible collapse over the next 200 to 1000 years. Because of its size (1.9 lakh square km), it contains enough water to raise the world sea level by more than half a metre. Recent study:

• A 2019 study had discovered a fast-growing cavity in the glacier sized roughly two-thirds the area of Manhattan. Then last week, the New York University study reported warm water at just two degrees above freezing point at Thwaites’s “grounding zone” or “grounding line”. • When glaciers melt and lose weight, they float off the land where they used to be situated. • When this happens, the grounding line retreats. That exposes more of a glacier’s underside to seawater, increasing the likelihood it will melt faster.

18

• This results in the glacier speeding up, stretching out, and thinning, causing the grounding line to retreat ever further. Hence both statements are correct. Source: https://vajiramias.com/current-affairs/thwaites- glacier/5e40e0f41d5def2722a25585/ Q25. With reference to the Pangolins, consider the following statements: 1. There are a total of two pangolin species across Africa and Asia. 2. The Indian Pangolin was categorised as critically endangered in IUCN Red List. Which of the statements given above is/are correct? a. 1 only b. 2 only c. Both 1 and 2 d. Neither 1 nor 2

Answer: b

Explanation:

Pangolins are scaly anteaters that are mammals of the order Pholidota.

• The eight species: There are a total of eight pangolin species across Africa and Asia. o Asian species: Sunda Pangolin, Philippine Pangolin, Chinese Pangolin and ‘Indian Pangolin’. o African species: Long-tailed Pangolin, Tree Pangolin, Giant Pangolin and the Ground Pangolin. • Characteristics: o They have large, protective keratin scales covering their skin, and they are the only known mammals with this feature. o They roll into a ball when threatened which can make them easy pickings for poachers. o Their diet consists of mainly ants and termites which they capture using their long tongues (A pangolin’s tongue is longer than its body). o Pangolins have no teeth; they chew with gravel and keratinous spines inside the stomach.

19

• According to the latest report released by TRAFFIC in March 2018, Pangolin is the most trafficked mammal in the world. Though hunted for its meat across the northeastern States and in central India, it is in huge demand for its scales in China. • IUCN status: The Chinese Pangolin was officially categorised as critically endangered in 2014, but it is believed to be extinct today. The Indian Pangolin, marked endangered that year, is now critically endangered and disappearing fast. • World Pangolin Day: The third Saturday of February is observed as the World Pangolin Day. Hence only statement 2 is correct. Source: https://vajiramias.com/current-affairs/pangolins/5e40ec0c1d5def2722a25784/ Q26. Which of the following are the steps taken by the Union Government to minimize import of steel? 1. Establishment of Steel Research and Technology Mission of India (SRTMI). 2. Imposition of Anti-Dumping Duty on a many steel products imports. 3. Steel Imports Monitoring System (SIMS) to monitor steel imports. Select the correct answer using the code given below: a. 1 only b. 1 and 2 only c. 2 and 3 only d. 1, 2 and 3

Answer: d

Explanation:

In order to minimize import of steel, the Government of India has taken following measures: 1. Domestically Manufactured Iron and Steel Products Policy (DMI&SP) has been notified with an objective to encourage consumption of domestically produced steel by Government organisations. 2. To meet the domestic demand for high performance steel, the Government, is facilitating R&D in the sector through the establishment of Steel Research and Technology Mission of India (SRTMI). 3. The Ministry is addressing the issue of quality of steel products that are produced as well as block import of sub-standard steel. It has notified Steel and Steel Products Quality Control (QC) Orders.

20

4. To prevent import due to unfair trade, the government has imposed Anti-Dumping Duty on a number of steel products imports. 5. To reduce dependence on imported steel scrap, the government has notified the Steel Scrap Recycling Policy. 6. The Government has recently introduced Steel Imports Monitoring System (SIMS) to monitor steel imports. Hence, option (d) is the correct answer. Source: https://vajiramias.com/current-affairs/steel-imports/5e422ad21d5def4f3bf8a997/ Q27. Which of the following projects come under Godavari and Kaveri River Linking Project? 1. Godavari (Inchampalli/Janampet) - Krishna (Nagarjunasagar) 2. Krishna (Nagarjunasagar) - Pennar (Somasila) 3. Pennar (Somasila) – Cauvery (Grand Anicut) Select the correct answer using the code given below: a. 1 and 2 only b. 1 and 3 only c. 2 and 3 only d. 1, 2 and 3

Answer: d

Explanation:

The draft Detailed Project Report (DPR) of Godavari – Cauvery link project consisting of 3 links viz., Godavari (Inchampalli/Janampet) - Krishna (Nagarjunasagar), Krishna (Nagarjunasagar) - Pennar (Somasila) and Pennar (Somasila) – Cauvery (Grand Anicut) link projects has been completed by National Water Development Agency (NWDA). As per the draft DPR, about 247 TMC can be diverted from to Nagarjunsagar dam(through lifting) and further south for meeting the demands of Krishna, Pennar and Cauvery basins. The stage of implementation of a project would be reached after DPR is prepared with the consensus of concerned States and the requisite statutory clearances are obtained. Hence, option (d) is the correct answer. Source: https://vajiramias.com/current-affairs/godavari-and-kaveri-river-linking- project/5e422cb91d5def4f3ffafe7f/ Q28. With reference to the Legislative Assembly of Delhi, consider the following statements:

21

1. The Delhi Legislative Assembly was first constituted in 1950. 2. Delhi Legislative Assembly was re-established through the Constitution (Sixty-ninth Amendment) Act, 1991. Which of the statements given above is/are correct? a. 1 only b. 2 only c. Both 1 and 2 d. Neither 1 nor 2

Answer: b

Explanation:

The Legislative Assembly of Delhi, also known as Delhi Vidhan Sabha, is a unicameral law making body of the National Capital Territory of Delhi, one of the eight union territories in India. The Delhi Legislative Assembly was first constituted in 1952 under the Government of Part C States Act, 1951. With enactment of States Reorganisation Act, 1956 Delhi was no longer a Part-C State and was made a Union Territory. Delhi Legislative Assembly was re-established through the Constitution (Sixty-ninth Amendment) Act, 1991, followed by the Government of National Capital Territory of Delhi Act, 1991 the Sixty-ninth Amendment to the Constitution of India. It declared the Union Territory of Delhi to be formally known as National Capital Territory of Delhi and also supplements the constitutional provisions relating to the Legislative Assembly and the Council of Ministers and related matters. The Legislative Assembly is selected for period of five years. Hence only statement 2 is correct. Source: https://vajiramias.com/current-affairs/delhi-assembly- elections/5e436fde1d5def4f3bf8de93/ Q29. Consider the following statements regarding juvenile justice system: 1. A juvenile in conflict with law is to be placed immediately under the care of the special juvenile police unit or a designated child welfare officer. 2. Once a child is produced before a Juvenile Justice Board (JJB), bail is the rule. Which of the statements given above is/are correct? a. 1 only b. 2 only

22

c. Both 1 and 2 d. Neither 1 nor 2

Answer: c

Explanation:

The Supreme Court has recently made it clear that the police have no right to detain children in conflict with law in a lockup or a jail. A juvenile in conflict with law, if apprehended, has to be placed immediately under the care of the special juvenile police unit or a designated child welfare officer. The child has to be produced before the Juvenile Justice Board (JJB). Once a child is produced before a JJB, bail is the rule. If for some reason bail is not granted, a child cannot be put behind bars. He has to be lodged either in an observation home or in a place of safety. The law is meant to protect children and not detain them in jail or keep them in police custody. The order came after the recent media reports about “children being detained in police custody and tortured in Delhi and Uttar Pradesh” in connection with the protests against the Citizenship (Amendments) Act. Hence both statements are correct. Source: https://vajiramias.com/current-affairs/detaining- children/5e44e0491d5def4f3eae0b33/ Q30. With reference to the Yaravirus, consider the following statements: 1. It only infects human cells. 2. It has been recently discovered in a lake in Brazil. Which of the statements given above is/are correct? a. 1 only b. 2 only c. Both 1 and 2 d. Neither 1 nor 2

Answer: b

Explanation:

In a lake in Brazil, researchers have recently discovered a virus called Yaravirus that has a “puzzling origin and phylogeny”. The Yaravirus infects amoeba and has genes that have not been described before, something that could challenge how DNA viruses are classified. Over 90% of the Yaravirus’s genome has not been observed before. Because of the Yaravirus’s small

23

size, it was unlike other viruses that infect amoeba and they named it as a tribute to Yara, the “mother of waters” in the mythological stories of the Tupi-Guarani indigenous tribes. The virus does not infect human cells. Although it is only now that the virus has been identified, the researchers believe that it has been present on Earth for ages. Hence only statement 2 is correct. Source: https://vajiramias.com/current-affairs/yaravirus/5e44e0da1d5def4f3eae0b43/ Q31. With reference to the recent Supreme Court judgment on criminalization of politics, consider the following statements: 1. The criminal history of candidates for the Assembly and Lok Sabha elections should be published in a local and a national newspaper as well as the parties’ social media handles. 2. It should mandatorily be published either within 48 hours of the selection of candidates or less than two weeks before the first date for filing of nominations, whichever is earlier. Which of the statements given above is/are correct? a. 1 only b. 2 only c. Both 1 and 2 d. Neither 1 nor 2

Answer: c

Explanation:

The Supreme Court recently ordered political parties to publish the entire criminal history of their candidates for the Assembly and Lok Sabha elections along with the reasons that goaded them to field suspected criminals over decent people. The information should be published in a local and a national newspaper as well as the parties’ social media handles. It should mandatorily be published either within 48 hours of the selection of candidates or less than two weeks before the first date for filing of nominations, whichever is earlier. It ordered political parties to submit compliance reports with the Election Commission of India within 72 hours or risk contempt of court action. The judgment is applicable to parties both at the Central and State levels. The published information on the criminal antecedents of a candidate should be detailed and include the nature of the offences, charges framed against him, the court concerned and the case number. Hence both statements are correct. Source: https://vajiramias.com/current-affairs/criminalisation-of- polity/5e4627731d5def4f3eae43a5/

24

Q32. ‘The Future of Earth, 2020’ report, recently seen in news, was released by the: a. European Union b. United Nations Framework Convention on Climate Change c. The Energy and Resources Institute d. None of the above Answer: d

Explanation:

‘The Future of Earth, 2020’ report was jointly released by the South Asia Future Earth Regional Office, Divecha Centre for Climate Change and the Indian Institute of Science. The report lists five global risks that have the potential to impact and amplify one another in ways that may cascade to create global systemic crisis. The five global risks are: failure of climate change mitigation and adaptation; extreme weather events; major biodiversity loss and ecosystem collapse; food crises; and water crises. Future Earth is an international sustainability research network. The report was prepared with the aim of reducing carbon footprint and halting global warming below 2 degree Celsius by 2050. Hence, option (d) is the correct answer. Source: https://vajiramias.com/current-affairs/the-future-of-earth-2020- report/5e4620a91d5def4f40a196de/ Q33. Which of the following pairs is/are correctly matched?

Select the correct answer using the code given below: a. 3 only b. 1 and 2 only c. 2 and 3 only d. 1, 2 and 3

25

Answer: a Explanation: Pair (1) is not correctly matched: The National Economic Council (ECNEC) of Bangladesh, chaired by Prime Minister Sheikh Hasina, has recently approved the proposal to enhance the capacity of the Mongla port. The Port of Mongla is located in Bagerhat District of Bangladesh. It lies 62 km north of the coastline. It is situated at the confluence of the Pasur River and the Mongla River. The Port is surrounded and protected by the Sundarban forest. Mongla is the second largest sea port in Bangladesh after Chittagong. India and Bangladesh had signed a MoU on the use of Chattogram and Mongla ports in 2015. Pair (2) is not correctly matched: Lombardy is a region in Northern Italy. Its capital, Milan, is a global hub of fashion and finance. It is the region hardest-hit by the coronavirus crisis in Italy. Pair (3) is correctly matched: Hubei province has been the epicentre of the outbreak of COVID- 2019 (the disease caused by the novel coronavirus). Hubei is a landlocked province in Central China. Its capital is Wuhan. The name of the province means "north of the lake", referring to its position north of Dongting Lake. The two major rivers of Hubei are the Yangtze River and its left , the Han River. The high-profile Three Gorges Dam is located in the province. The Yangtze River enters Hubei from the west via the Three Gorges Dam. Hence, option (a) is the correct answer. Source: https://vajiramias.com/current-affairs/mongla-port/5e51d1a91d5def58ff370b01/ https://vajiramias.com/current-affairs/hubei/5e476d9e1d5def4f3c89f12b/ Q34. With reference to the FATF, consider the following statements: 1. FATF Black list consists of the countries that use terrorism as a state policy to conduct its proxy wars. 2. FATF Grey List consists of countries that offer safe haven for supporting terror funding and money laundering. Which of the statements given above is/are correct? a. 1 only b. 2 only c. Both 1 and 2 d. Neither 1 nor 2

Answer: b Explanation:

26

The Financial Action Task Force (FATF) is an inter-governmental body established in 1989 by the Ministers of its Member jurisdictions. The objectives of the FATF are to set standards and promote effective implementation of legal, regulatory and operational measures for combating money laundering, terrorist financing and other related threats to the integrity of the international financial system. The FATF monitors the progress of its members in implementing necessary measures, reviews money laundering and terrorist financing techniques and counter- measures, and promotes the adoption and implementation of appropriate measures globally. The FATF’s decision making body, the FATF Plenary, meets three times a year. It has recently put Pakistan on its grey list. "FATF grey list" is called as "Other monitored jurisdictions". FATF has 2 types of lists: a. Black List: Countries known as Non-Cooperative Countries or Territories (NCCTs) are put in the blacklist. These countries support terror funding and money laundering activities. The FATF revises the blacklist regularly, adding or deleting entries. Hence, statement (a) is the not correct. b. Grey List: Countries that are considered safe haven for supporting terror funding and money laundering are put in the FATF grey list. This inclusion serves as a warning to the country that it may enter the blacklist. Hence, statement (b) is the correct. Consequences of being in the FATF Grey list: 1. Economic sanctions from IMF, World Bank, ADB 2. Problem in getting loans from IMF, World Bank, ADB and other countries 3. Reduction in international trade 4. International boycott Source: https://vajiramias.com/current-affairs/fatf-plenary-19-21-february- 2020/5e5335e51d5def661eca68a7/ Q35. With reference to the Jet streams, consider the following statements: 1. They are narrow bands of strong winds that flow over thousands of kilometres from west to east. 2. In India, the Tropical jet stream influences the formation and duration of the winter monsoon. Which of the statements given above is/are correct? a. 1 only b. 2 only c. Both 1 and 2

27

d. Neither 1 nor 2

Answer: a

Explanation:

Statement 1 is correct: Jet streams are narrow bands of strong winds that flow over thousands of kilometres from west to east. Major jet streams are found near the upper levels of the atmosphere, around 9 to 16 km from the earth’s surface, and can reach speeds of over 320 kph. The jet streams shift to the north or south depending on the season. During winters, the wind current is the strongest. They are also closer to the Equator during winter. The major jet streams are the Polar Front, Subtropical, and Tropical jet streams. Statement 2 is incorrect: In India, the Tropical jet stream influences the formation and duration of the summer monsoon. Source: https://vajiramias.com/current-affairs/jet-stream/5e48ab1a1d5def4f3c8a2a48/ Q36. With reference to the private member’s Bill, consider the following statements: 1. Recently, a private member’s bill has been passed in the Parliament of India for the first time in its history. 2. It can be introduced and discussed on any day during the session. Which of the statements given above is/are correct? a. 1 only b. 2 only c. Both 1 and 2 d. Neither 1 nor 2

Answer: d

Explanation:

An MP who is not a minister is a private member. While both private members and ministers take part in the lawmaking process, Bills introduced by private members are referred to as private member’s Bills and those introduced by ministers are called government Bills. While a government Bill can be introduced and discussed on any day, a private member’s bill can only be introduced and discussed on Fridays. Government Bills are backed by the government and

28

also reflect its legislative agenda. Individual MPs may introduce private member’s Bill to draw the government’s attention to what they might see as issues requiring legislative intervention. The admissibility of a private Bill is decided by the Chairman in the case of the Rajya Sabha and the Speaker in the case of the Lok Sabha. Before the Bill can be listed for introduction, the Member must give at least a month’s notice, for the House Secretariat to examine it for compliance with constitutional provisions and rules on legislation. No private member’s Bill has been passed by Parliament since 1970. To date, Parliament has passed 14 such Bills, six of them in 1956. Hence both statements are incorrect. Source: https://vajiramias.com/current-affairs/private-members- bill/5e48ac651d5def4f3ffc25c1/ Q37. Which of the following pairs is/are correctly matched?

Select the correct answer using the code given below: a. 1 only b. 1 and 2 only c. 2 and 3 only d. 1, 2 and 3 Answer: b Explanation: Pair (1) is correctly matched: Kambala is an annual Buffalo Race held in the Tulu region of Karnataka (districts of Dakshina and Udupi). The contest generally takes place between two pairs of buffaloes, each pair race in wet rice fields, controlled by a whip-lashing farmer. The Kambala season generally starts in November and lasts till March during which races are held under the banner of Kambala Samithi (Kambala Association). Some farmers also run their buffalo in non-competitive ritualistic manner for thanksgiving for protecting their animals from diseases.

29

Pair (2) is correctly matched: Jallikattu, a popular bull taming sport held during , is set to take place in Madurai district, Tamil Nadu. The three main venues where the sport will take place in are Palamedu, Alanganallur and Avaniapuram. Jallikattu is a dated tradition. An ancient reference to bull taming is found in a seal discovered at Mohenjodaro, which is dated between 2,500 BC and 1,800 BC. The sport was called Eru thazuval or “embracing the bull”. Pair (3) is not correctly matched: Bail Gadi Sharyat held in the state of Maharashtra. It is an essential feature during the annual village festival called ‘Jatra’. Hence, option (b) is the correct answer. Source: https://vajiramias.com/current-affairs/kambala/5e48ad021d5def4f3bf9cc0b/ Q38. With reference to the recusal by judge, consider the following statements: 1. It usually takes place when a judge has a conflict of interest or has a prior association with the parties in the case. 2. The reasons for recusal are not disclosed in an order of the court. Which of the statements given above is/are correct? a. 1 only b. 2 only c. Both 1 and 2 d. Neither 1 nor 2

Answer: c

Explanation:

Recusal usually takes place when a judge has a conflict of interest or has a prior association with the parties in the case. There are no written rules on the recusal of judges from hearing cases listed before them in constitutional courts. It is left to the discretion of a judge. The reasons for recusal are not disclosed in an order of the court. Some judges orally convey to the lawyers involved in the case their reasons for recusal, many do not. A recusal inevitably leads to delay. The case goes back to the Chief Justice, who has to constitute a fresh Bench. Hence both statements are correct. Source: https://vajiramias.com/current-affairs/recusals/5e49e5541d5def4f3eaf3fe9/ Q39. The initiative, PIC is a programme to research on: a. Coronavirus

30

b. Defence c. Nano technology d. Indigenous Cows Answer: d Explanation: The Government of India has recently unveiled a programme to research on ‘indigenous’ cows. The initiative is called SUTRA PIC or Scientific Utilisation Through Research Augmentation-Prime Products from Indigenous Cows. The objective is Scientific research will be carried out milk products derived from Indian indigenous cows. It has five themes: o Uniqueness of Indigenous Cows, o Prime-products from Indigenous Cows for Medicine & Health, o Prime-products from Indigenous Cows for Agricultural Applications, o Prime-products from Indigenous Cows for Food & Nutrition and o Prime-products from indigenous cows-based utility items. To be funded by multiple scientific ministries, the initiative is led by the Department of Science and Technology (DST). Researchers from academic organisations as well as NGOs active in India were invited to apply for funding. Hence, option (d) is the correct answer. Source: https://vajiramias.com/current-affairs/sutra-pic/5e4b45401d5def4f3ffcec95/ Q40. Which of the following pairs is/are correctly matched?

Select the correct answer using the code given below: a. 1 only b. 1 and 2 only c. 2 and 3 only d. 1, 2 and 3

31

Answer: c Explanation: Pair (1) is not correctly matched: Pakke Tiger Reserve, also known as Pakhui Tiger Reserve, is a Project Tiger reserve. It is located in the East Kameng district of Arunachal Pradesh. Towards the south and south-east, the sanctuary adjoins Nameri National Park of Assam. To the east lies the Pakke River and to the west, the park is bounded by the Bhareli or Kameng River. It falls within the Eastern Himalaya Biodiversity Hotspot. Pair (2) is correctly matched: Kawal Tiger Reserve is located in . Govt of India declared as Tiger Reserve in 2012. The reserve is the oldest sanctuary in the northern Telangana region of the state. It is well known for its abundant flora and fauna. The reserve is catchment for the rivers Godavari and Kadam, which flow towards the south of it. Pair (3) is correctly matched: Wayanad Wildlife Sanctuary is an animal sanctuary in Wayanad, . The sanctuary is an integral part of the Nilgiri Biosphere Reserve. It is bounded by protected area network of Nagarhole and Bandipur of Karnataka in the northeast, and on the southeast by Mudumalai of Tamil Nadu. Wayanad Wildlife Sanctuary is home to more than half the number of tigers present in Kerala. Hence, option (c) is the correct answer. Source: https://vajiramias.com/current-affairs/pakke-tiger- reserve/5e5472931d5def66248cee50/ https://vajiramias.com/current-affairs/wayanad-wildlife- sanctuary/5e4b40721d5def4f3ffcec33/ Q41. With reference to the Bodo language, consider the following statements: 1. It is not listed in the Eighth Schedule of the Constitution. 2. The 2020 Accord makes Bodo the associate official language throughout Assam. Which of the statements given above is/are correct? a. 1 only b. 2 only c. Both 1 and 2 d. Neither 1 nor 2

Answer: b

32

Explanation:

The Bodo language is one of the key thrust areas in the Bodo Accord. Estimated to have 1.5 million speakers (Census 2011), Bodo is listed in the Eighth Schedule of the Constitution. It is spoken in Assam, where the Bodo tribe constitutes about 5-6% of the population, and in Arunachal Pradesh, Nagaland, Meghalaya, and West Bengal. While Bodo is officially written in the Devanagri script, the language has a history of having been written in at least three different scripts — until in 1974, the Government recognised Devanagari as its official script. The 2020 Accord makes Bodo the associate official language throughout Assam. The new Accord also promises to establish a separate directorate for Bodo medium schools, provincialise schools and colleges in the BTAD (Bodoland Territorial Autonomous District) and establish a Cultural Complex-cum-Centre of Excellence named after the late social activist Bodofa Upendranath in Kokrajhar for protection and promotion of the language. Hence only statement 2 is correct. Source: https://vajiramias.com/current-affairs/bodo-language/5e4c95da1d5def4f3eafd33f/ Q42. With reference to the Farmer Producer Organizations (FPOs), consider the following statements: 1. FPOs ensure economies of scale for farmers. 2. FPOs are to be used to promote under "One District One Product" cluster to promote specialization. Which of the statements given above is/are correct? a. 1 only b. 2 only c. Both 1 and 2 d. Neither 1 nor 2

Answer: c

Explanation:

The Cabinet Committee on Economic Affairs (CCEA) has recently approved a scheme for "Formation and Promotion of Farmer Producer Organizations (FPOs)" to form and promote 10,000 new FPOs. The Cabinet Committee on Economic Affairs (CCEA) has given its approval for 10,000 FPOs to be formed in five years period from 2019-20 to 2023-24 to ensure economies of scale for farmers. Support to each FPO be continued for 5 years from its year of inception. Initially there will be three implementing Agencies to form and promote FPOs, namely Small

33

Farmers Agri-business Consortium (SFAC), National Cooperative Development Corporation (NCDC) and National Bank for Agriculture and Rural Development (NABARD). FPOs will be formed and promoted through Cluster Based Business Organizations (CBBOs) engaged at the State/Cluster level by implementing agencies. FPOs will be promoted under "One District One Product" cluster to promote specialization and better processing, marketing, branding & export by FPOs. Through formation of FPOs, farmers will have better collective strength for better access to quality input, technology, credit and better marketing access through economies of scale for better realization of income. Hence both statements are correct. Source: https://vajiramias.com/current-affairs/farmer-producer-organizations- fpos/5e4dd8881d5def4f3c8b79b5/ Q43. With reference to the Soil Health Card scheme, consider the following statements: 1. Its objective is to provide a basis to address nutritional deficiencies in fertilization practises. 2. The International year of soils was celebrated in 2015, the same year India’s Soil Health Card (SHC) scheme was launched. Which of the statements given above is/are correct? a. 1 only b. 2 only c. Both 1 and 2 d. Neither 1 nor 2

Answer: c

Explanation:

The Soil Health Card Day was observed on February 19 to commemorate the day Soil Health Card Scheme was launched by the PM Modi on February 19, 2015 at Suratgarh, Rajasthan. The objectives of the Soil Health Card scheme are to issue soil health cards to farmers every 2 years so as to provide a basis to address nutritional deficiencies in fertilization practises. Soil health card provides information to farmers on nutrient status of their soil along with recommendation on appropriate dosage of nutrients to be applied for improving soil health. Soil testing is developed to promote soil test based on nutrient management. Soil testing reduces cultivation cost by application of right quantity of fertilizer. A 2017 study by the National Productivity Council (NPC) found that the SHC scheme has promoted sustainable farming and led to a decrease of use of chemical fertilizer application in the range of 8-10%. Besides, overall increase in the yield of crops to the tune of 5-6% was reported due to application of fertilizer and micro nutrients as per recommendations available in the Soil Health

34

Cards. The International year of soils was celebrated in 2015, the same year India’s Soil Health Card (SHC) scheme was launched. Hence both statements are correct. Source: https://vajiramias.com/current-affairs/soil-health-card-shc- scheme/5e4dd9151d5def4f3bfb2bb5/ Q44. With reference to the "Pradhan Mantri Fasal Bima Yojana (PMFBY)", consider the following statements: 1. It is entirely centrally sponsored scheme but its execution remains in the hands of the states. 2. Enrolment under the Scheme to be made voluntary for all farmers for both schemes. Which of the statements given above is/are correct? a. 1 only b. 2 only c. Both 1 and 2 d. Neither 1 nor 2

Answer: b

Explanation:

The Union Cabinet has recently approved revamping of "Pradhan Mantri Fasal Bima Yojana (PMFBY)" and "Restructured Weather Based Crop Insurance Scheme (RWBCIS)" to address the existing challenges in implementation of Crop Insurance Schemes. It is proposed to modify following provisions of PMFBY and RWBCIS:

• Allocation of business to Insurance Companies to be done for three years (Both PMFBY/RWBCIS). • Option shall be given to States/UTs to choose Scale of Finance or district level Value of Notional Average Yield (NAY) i.e. Minimum Support Price (MSP) as Sum Insured for any district crop combination (Both PMFBY/RWBCIS). • Central Subsidy under PMFBY/RWBCIS to be limited for premium rates upto 30% for unirrigated areas/crops and 25% for irrigated areas/crops. Districts having 50% or more irrigated area will be considered as irrigated area/district (Both PMFBY/RWBCIS). • Enrolment under the Scheme to be made voluntary for all farmers (Both PMFBY/RWBCIS). • Central Share in Premium Subsidy to be increased to 90% for North Eastern States from the existing sharing pattern of 50:50 (Both PMFBY/RWBCIS). • Provisioning of at least 3% of the total allocation for the Scheme to be made by Government of India and Implementing State Governments for administrative expenses. 35

This shall be subject to an upper cap fixed by DAC&FW for each State (Both PMFBY/RWBCIS). • These changes are proposed to be implemented from Kharif’ 2020 Season throughout the Country. Hence only statement 2 is correct. Source: https://vajiramias.com/current-affairs/crop-insurance- schemes/5e4dd9fb1d5def4f3eb01642/ Q45. Tilhan Mission, recently seen in news, is related to: a. Oilseed production b. Supercomputers c. Indigenous defence manufacturing d. Nutrition of children Answer: a Explanation: Union Agriculture Minister recently announced that the government will launch Tilhan Mission to make the country self-reliant in oilseed production. Total Oilseeds production in the country during 2019-20 is estimated at 34.19 million tonnes which is higher by 2.67 million tonnes than the production of 31.52 million tonnes during 2018-19. Further, the production of oilseeds during 2019-20 is higher by 4.54 million tonnes than the average oilseeds production. India is the fourth largest vegetable oil economy in the world after the USA, China and Brazil. Hence, option (a) is the correct answer. Source: https://vajiramias.com/current-affairs/tilhan-mission/5e5077511d5def55e5a32287/ Q46. With reference to the UN Convention on Migratory Species, consider the following statements: 1. Appendix I of UN Convention on Migratory Species lists migratory species that are endangered. 2. Great Indian Bustard, Asian Elephant and Bengal Florican was recently moved from Appendix I to Appendix II of UN Convention on migratory species. Which of the statements given above is/are correct? a. 1 only b. 2 only

36

c. Both 1 and 2 d. Neither 1 nor 2

Answer: a

Explanation:

India’s proposal to include Great Indian Bustard, Asian Elephant and Bengal Florican in Appendix I of UN Convention on migratory species was unanimously accepted at the 13th Conference of the Parties to the Convention on Migratory Species (CMS) recently held in Gandhinagar. Asian Elephant:

• The Government of India has declared Asian elephant/Indian elephant (Elephas maximus indicus) as National Heritage Animal. • Indian elephant is also provided highest degree of legal protection by listing it in Schedule I of the Wildlife (Protection) Act, 1972. • The challenges confronting Asian elephant conservation in most elephant Range States are habitat loss and fragmentation, human elephant conflict, and poaching and illegal trade of elephants. Great Indian Bustard:

• The Great Indian Bustard is a Critically Endangered species with a small population of about 100–150 individuals that is largely restricted to Thar desert in Rajasthan, India. • The species has disappeared from 90% of this range; their population has reduced by 90% within 50 years (six generations); and their threats are expected to increase in future. Bengal Florican:

• The Bengal Florican, a critically endangered species, exhibits transboundary movements. Its migration exposes it to threats such as land use changes and collision with power transmission line at boundary area of India-Nepal. • As a result, populations have declined and the species no longer breeds outside Protected Areas in the , except in a few areas of Assam. Convention on the Conservation of Migratory Species of Wild Animals. Also known as Bonn Convention (because the intergovernmental conference where the Convention was negotiated took place in Bonn in 1979. The species covered by CMS are Mammals, birds, reptiles, fish and one insect are listed on the Convention’s two Appendices – Appendix I lists migratory species that are endangered. Appendix II lists migratory species which have an unfavourable conservation status and which require international agreements for their conservation and management. Hence only statement 1 is correct.

37

Value Addition:

• The Great Indian Bustard was the mascot of COP13. The CMS COP 13 Logo was also inspired by the traditional ‘KOLAM’ from southern India, which has a profound significance in the context of living in harmony with nature. • The theme for the CMS COP 13 was “Migratory species connect the planet and together we welcome them home”. • Ten new species were added to CMS Appendices at COP13. o Seven species were added to Appendix I: Asian Elephant, Jaguar, Great Indian Bustard, Bengal Florican, Little Bustard, Antipodean Albatross and the Oceanic White-tip Shark. o Three species were added to Appendix II: Urial, Smooth Hammerhead Shark and the Tope Shark. • New and extended Concerted Actions with targeted conservation plans were agreed for 14 species. • CMS COP13 also adopted the Gandhinagar Declaration, which calls for the concept of ‘ecological connectivity’ to be integrated and prioritized in the new Framework, which is expected to be adopted at the UN Biodiversity Conference in October. • The first ever report on the Status of Migratory Species, presented to CMS COP13, shows that despite some success stories, the populations of most migratory species covered by CMS are declining. • Three CMS Ambassadors - for terrestrial, avian, and aquatic species - were named at the relaunch of the CMS Ambassadors Programme: Ian Redmond OBE (for terrestrial), Sacha Dench (for avian), and Indian actor Randeep Hooda (for aquatic). • Seven new Migratory Species Champions including the Government of India were named at a special high-level event organized on the sidelines of the conference. • India, as COP13 host, will assume the role of COP Presidency for the next three years. • The Conference kicked off “Super Year for Environment”, which will include a UN Summit in September and culminate in the UN Biodiversity Conference at the end of 2020, when a new global biodiversity strategy for the next decade will be adopted - the Post-2020 Global Biodiversity Framework. Source: https://vajiramias.com/current-affairs/appendix-i-of-un-convention-on-migratory- species/5e507a511d5def55e864ad97/ Q47. Which of the following pairs is/are correctly matched?

38

Select the correct answer using the code given below: a. 3 only b. 1 and 2 only c. 2 and 3 only d. 1, 2 and 3

Answer: a

Explanation:

Pair (1) is not correctly matched: is a traditional theatre form that combines dance, music, dialogue, costume, make-up, and stage techniques with a unique style and form. Yakshagana literally means the song (gana) of the yaksha (nature spirits). It developed in Udupi, in the state of Karnataka. It is popular in the Karnataka districts of Dakshina Kannada, Kasaragod, Udupi, Uttara Kannada and Shimoga. This folk art is believed to have originated somewhere in between the 10th and 16th century. Yakshagana is strongly influenced by the Vaishnava movement. Its stories are mainly drawn from , , Bhagavata and other Hindu epics. A typical Yakshagana performance consists of background music played by a group of musicians (known as the himmela); and a dance and dialog group (known as the mummela), who together enact poetic epics on stage. Yakshagana is traditionally presented from dusk to dawn.

• The tenkutittu style is prevalent in Dakshina Kannada. Tenkutittu is noted for its incredible dance steps; its high flying dance moves; and its extravagant rakshasas (demons).

39

• The Badagutittu style is prevalent in Uttara Kannada District and places more emphasis on facial expressions, matugarike (dialogues), and dances appropriate for the character depicted in the episode. Pair (2) is not correctly matched: The dance form was introduced in the 15th century A.D by the great Vaishnava saint and reformer of Assam, Mahapurusha Sankaradeva as a powerful medium for propagation of the Vaishnava faith. The dance form evolved and expanded as a distinctive style of dance later on. This neo-Vaishnava treasure of Assamese dance and drama has been, for centuries, nurtured and preserved with great commitment by the Sattras i.e. Vaishnava maths or monasteries. Pair (3) is correctly matched: Kerala is the home of several traditional dance and dance - drama forms, the most notable being . It is an art which has evolved from many social and religious theatrical forms which existed in the southern region in ancient times. Chakiarkoothu, Koodiyattam, Krishnattam andRamanattam are few of the ritual performing arts of Kerala which have had a direct influence on Kathakali in its form and technique. Hence, option (a) is the correct answer. Source: https://vajiramias.com/current-affairs/yakshagana/5e5339a31d5def6620f27057/ Q48. With reference to the New Delhi International Arbitration Centre (NDIAC) Act 2019, consider the following statements: 1. New Delhi International Arbitration Centre (NDIAC) is to be headed by a Chairperson, who has been the Chief Justice of India. 2. Other members are eminent persons having experience in institutional arbitration, both domestic and international. Which of the statements given above is/are correct? a. 1 only b. 2 only c. Both 1 and 2 d. Neither 1 nor 2

Answer: b

Explanation:

40

Draft New Delhi International Arbitration Centre (NDIAC) Rules have been issued for public consultation with the timeline of submission of comments by 14th March, 2020.

• The New Delhi International Arbitration Centre (NDIAC), Act 2019 was enacted to establish and make NDIAC a hub for institutional arbitration and to declare it as an institution of national importance. • NDIAC will be headed by a Chairperson, who has been a Judge of the Supreme Court or a Judge of a High Court or an eminent person appointed by the Central Government in consultation with the Chief Justice of India. • Besides, it will also have two Full-time or Part-time Members from amongst eminent persons having experience in institutional arbitration, both domestic and international. • In addition, one representative of a recognized body of commerce and industry shall be nominated on rotational basis as a Part-time Member. • The Secretary, Department of Legal Affairs, Ministry of Law & Justice; Financial Adviser nominated by Department of Expenditure, Ministry of Finance and Chief Executive Officer, NDIAC will be ex-officio Members. • Section 23 of the Act provides for the Secretariat to the Centre inter-alia comprising Registrar, Counsel and other officers & employees etc. In this regard, the Department of Legal Affairs has prepared the following draft Rules:

• The New Delhi International Arbitration Centre (the terms and conditions and the salary and allowances payable to the Chairperson and Full-time Members) Rules 2020. • The New Delhi International Arbitration Centre (the travelling and other allowances payable to Part-time Members) Rules 2020. • The New Delhi International Arbitration Centre (the number of officers and employees of the Secretariat of the Centre) Rules 2020. • The New Delhi International Arbitration Centre (the qualifications, experience, method of selection and the functions of the Registrar, Counsel and other officers and employees of the Centre) Rules 2020. Hence only statement 2 is correct. Source: https://vajiramias.com/current-affairs/draft-new-delhi-international-arbitration- centre-ndiac-rules/5e54717d1d5def66248cee3f/ Q49. With reference to the Blue Dot network, consider the following statements: 1. It is a multi-stakeholder initiative to promote high-quality, trusted standards for global infrastructure development”.

2. It is an initiative led by China.

Which of the statements given above is/are correct?

41

a. 1 only b. 2 only c. Both 1 and 2 d. Neither 1 nor 2

Answer: a

Explanation:

The Blue Dot network is “a multi-stakeholder initiative to bring together governments, the private sector and civil society to promote high-quality, trusted standards for global infrastructure development”. The proposal, which is part of the US’s Indo-Pacific strategy, will evaluate projects on various parameters, including level of public consultation, transparency in funding, debt traps and basic environment norms. Projects that meet the norms will get a “blue dot”, which will enable them to attract private funding and not have to depend on state- funding alone. The US, which is leading the proposal, has already got Japan and Australia as partners in this idea. The proposal is aimed at countering Chinese President Xi Jinping’s ambitious One Belt One Road initiative. Under Belt and Road Initiative (BRI), China’s government and state-owned enterprises finance international projects by providing logistical support — from concrete and steel to workers and cash. This approach, however, has been labelled by some experts as “debt- trap diplomacy”. Blue Dot will be about “supporting alternatives to predatory lending” by facilitating foreign investment in projects that come under this network. Hence only statement 1 is correct. Source: https://vajiramias.com/current-affairs/blue-dot- network/5e5473521d5def6620f2bb23/ Q50. InSight lander mission, often mentioned in news, is an initiative of: a. National Aeronautics and Space Administration b. Japan Aerospace Exploration Agency c. Indian Space Research Organisation d. China National Space Administration Answer: a Explanation:

42

NASA recently published a set of six papers to reveal findings of NASA’s InSight lander mission. InSight is an acronym for “Interior Exploration using Seismic Investigations, Geodesy and Heat Transport”. • This mission is part of “NASA's Discovery Program.” • It is the first mission to study in-depth the "inner space" of Mars: its crust, mantle, and core. • InSight won’t be looking for life on Mars. Instead it will study its insides i.e. it seeks to understand the formation and evolution of terrestrial planets through investigation of the interior structure and processes of Mars. • The mission successfully landed at Elysium Planitia on Mars in November 2018. Key findings:

• Trembles: Mars trembles more often than expected, but also more mildly. Mars doesn’t have tectonic plates like Earth, but it does have volcanically active regions that can cause rumbles. • Magnetism: At a Martian site called Homestead hollow, the magnetic signals are 10 times stronger than what was predicted earlie r. • Dust devils: It has detected/felt (but hasn’t seen yet) thousands of passing whirlwinds, which are called dust devils when they pick up grit and become visible. Hence, option (a) is the correct answer. Source: https://vajiramias.com/current-affairs/insight-mars- lander/5e586e251d5def6623a265e4/ Q51. With reference to the Suposhit Maa Abhiyan, consider the following statements: 1. It aims to provide nutritional support to pregnant women and adolescent girls. 2. Under this Abhiyan, all women would be given food items for one year. Which of the statements given above is/are correct? a. 1 only b. 2 only c. Both 1 and 2 d. Neither 1 nor 2 Answer: a Explanation: The Lok Sabha Speaker recently launched "Suposhit Maa Abhiyan" to provide nutritional support to pregnant women and adolescent girls in Kota. Keeping newborn and pregnant women healthy is the main objective of the programme '. Under this Abhiyan, 1,000 women

43

would be given food items for one month. At the same time, the health of the child, including medical examination, blood tests, medicines, delivery, would be covered. The identified women would be required to register on a website for adoption. Only one pregnant woman would be adopted from a family. In the first phase of the campaign, 1,000 kits of 17 kg balanced diet each were provided to 1,000 pregnant women. Hence only statement 1 is correct. Hence, option (a) is the correct answer. Source: https://vajiramias.com/current-affairs/suposhit-maa- abhiyan/5e5b1caa1d5def6620f3d7a3/ Q52. Consider the following pairs:

Which of the following pairs is/are correctly matched? a. 1, 2 and 3 b. 1 and 2 only c. 2 only d. None of the above Answer: d Explanation: Pair (1) is not correctly matched: Mahakoshal region includes parts of UP, Madhya Pradesh and Jharkhand. It is known to be potentially mineral-rich. The land is mainly forest area and inhabited mostly by tribals and members of backward classes. Pair (2) is not correctly matched: is a geographical and cultural region divided between the states of Uttar Pradesh and Madhya Pradesh. Jhansi and Khajuraho are well known places in Bundelkhand. Pair (3) is not correctly matched: is the north-eastern region of Maharashtra, comprising Division and Amravati Division. , a notified National Geo-heritage Monument is located in of Vidarbha region.

44

Hence, option (d) is the correct answer. Source: https://vajiramias.com/current-affairs/bundelkhand- expressway/5e5b1bcd1d5def6620f3d79a/ Q53. With reference to the “Red snow”, consider the following statements: 1. It is the presence of the algae that gives the red color. 2. It melts faster than the ordinary snow, which is a concern for climate change. Which of the statements given above is/are correct? a. 1 only b. 2 only c. Both 1 and 2 d. Neither 1 nor 2 Answer: c Explanation: “Red snow” is a phenomenon that has been known since ancient times. It is the algae that give the snow its red tinge. This alga species, Chlamydomonas Chlamydomonasnivalis, exists in snow in the polar and glacial regions, and carries a red pigment to keep itself warm. Now, it raises concerns about climate change. The red snow causes the surrounding ice to melt faster. The more the algae packed together, the redder the snow. And the darker the tinge, the more the heat absorbed by the snow. Subsequently, the ice melts faster which is bad for glaciers. These algae change the snow’s albedo — which refers to the amount of light or radiation the snow surface is able to reflect back. Changes in albedo lead to more melting. Hence both statement 1 and 2 are correct. Hence, option (c) is the correct answer. Source: https://vajiramias.com/current-affairs/red-snow/5e5b1c0b1d5def6623a2dde3/ Q54. Which of the following best describes RaIDer-X, recently seen in the news? a. It is a new explosive detection device. b. They are snow and ice features formed by erosion and characterized by bowl-shaped depressions. c. It is a spacecraft which will travel to a near-Earth asteroid, called Bennu. d. None of the above

45

Answer: a Explanation: RaIDer-X, a new explosive detection device, was recently unveiled at the National Workshop on Explosive Detection (NWED-2020) in Pune. RaIDer-X has been co-developed by High Energy Materials Research Laboratory (HEMRL) Pune and Indian Institute of Science, Bangalore. HEMRL Pune is a premier laboratory of DRDO. RaIDer-X has the capability to detect explosives from a stand-off distance. The data library can be built in the system to expand its capability to detect a number of explosives in pure form as well as with the contaminants. Bulk explosive in concealed condition can also be detected by the device. Hence, option (a) is the correct answer. Hence, option (a) is the correct answer. Source: https://vajiramias.com/current-affairs/raider-x/5e5c77a31d5def6620f45fde/ Q55. Consider the following pairs:

Which of the following pairs is/are correctly matched? a. 1 only b. 1 and 2 only c. 2 and 3 only d. 1 and 3 only Answer: d Explanation: Pair (1) is correctly matched: is a hill station and valley region in the southeastern Indian state of . It's surrounded by the thick forests of the mountain range.

46

Pair (2) is not correctly matched: The Lidder Valley or Liddar Valley in the union territory of Jammu and Kashmir, is a Himalayan sub-valley that forms the southeastern corner of the . The Lidder River flows down the valley. Pair (3) is correctly matched: Nubra Valley is in the Indian union territory of . Its inhabited areas form a tri-armed valley cut by the Nubra and Shyok rivers. Hence, option (d) is the correct answer. Source: https://vajiramias.com/current-affairs/araku-utsav/5e5c76361d5def6623a3708c/ Q56. With reference to the Laboratory for Conservation of Endangered Species (LaCONES), consider the following statements: 1. Its objective is to use modern biotechnologies for conservation of endangered wildlife. 2. It is India’s only dedicated laboratory for conservation of endangered species. Which of the statements given above is/are correct? a. 1 only b. 2 only c. Both 1 and 2 d. Neither 1 nor 2

Answer: c

Explanation:

Statement 1 is correct: LaCONES is an acronym for The Laboratory for the Conservation of Endangered Species. It is a dedicated Laboratory of CSIR’s Centre for Cellular and Molecular Biology (CCMB) in Hyderabad. Objective is to use modern biotechnologies for conservation of endangered wildlife (It supports both the measures of conservation i.e. in situ and ex situ measures). Statement 2 is correct: CCMB-LaCONES is the only laboratory in India that has developed methods for collection and cryopreservation of semen and oocytes from wildlife and successfully reproducing endangered blackbuck, spotted deer and Nicobar pigeons. Through this work, it has established Genetic Resource Bank for Indian wildlife. Project LaCONES was established in 1998 with support from (i) Dept. of Biotechnology (DBT), Govt. of India, (ii) Central Zoo Authority of India (CZA), Delhi, (iii) Council of Scientific and Industrial Research (CSIR) and (iv) Government of Andhra Pradesh.

47

The laboratory was itself established in 2007. It is India’s only dedicated laboratory for conservation of endangered species. Hence, option (c) is the correct answer. Source: https://vajiramias.com/current-affairs/laboratory-for-conservation-of-endangered- species-lacones/5e5c78cc1d5def66248ea055/ Q57. With reference to the National Security Guard (NSG), consider the following statements: 1. It has permanent infrastructure at four places in India which includes Kolkata, , and Hyderabad. 2. It was raised in 1993, following the assassination of Rajiv Gandhi. Which of the statements given above is/are correct? a. 1 only b. 2 only c. Both 1 and 2 d. Neither 1 nor 2

Answer: a

Explanation:

Statement 1 is correct: Union Home Minister recently inaugurated the National Security Guard (NSG) Regional Hub campus at Kolkata. This latest NSG complex has become a model regional hub of the NSG, which will help in honing the professional acumen of the NSG commandos and also contribute significantly in capacity building of the first responders, the Police forces of the States. The area of responsibility of this Hub comprises of West Bengal, Bihar, Jharkhand & the entire North East. The Kolkata hub is the fourth to have permanent infrastructure after Mumbai, Chennai and Hyderabad. Statement 2 is incorrect: National Security Guard (NSG) which comprises of Indian Army and CAPFs, has multi- dimensional responsibility of countering terrorist attacks/hijack attempts and also providing proximate security. Parent agency is Ministry of Home Affairs (MHA). It was raised in 1984, following Operation Blue Star and the assassination of Indira Gandhi. It has been formed under the National Security Guard Act, 1986. According to Ministry of Home Affairs website, it is one of the 7 Central Armed Police Forces (CAPF). Headquarters is at New Delhi.

48

Hence, option (a) is the correct answer. Source: https://vajiramias.com/current-affairs/national-security-guard-nsg-regional- hub/5e5c76e31d5def6623a370aa/ Q58. With reference to the Ancient Monuments and Archaeological Sites and Remains Act, 1958, consider the following statements: 1. The Act bans construction within 1000 metres of a centrally protected monument and regulated construction within 1000-2000 metres. 2. The Act protects monuments and sites that are over 100 years old. Which of the statements given above is/are correct? a. 1 only b. 2 only c. Both 1 and 2 d. Neither 1 nor 2

Answer: b

Explanation: The Ancient Monuments and Archaeological Sites and Remains Act, 1958 aims to provide for the preservation of ancient and historical monuments and archaeological sites and remains of national importance, for the regulation of archaeological excavations and for the protection of sculptures, carvings and other like objects. There is ban on construction within 100 metres of a Centrally protected monument and regulated construction within 100-200 metres under the Ancient Monuments and Archaeological Sites and Remains Act, 1958. The Act protects monuments and sites that are over 100 years old.

The Government of India is planning to conduct a review of monuments under the Archaeological Survey of India (ASI) and the ones protected by the State governments. The list of the Centrally protected monuments had not seen a substantial increase in many years, and important sites under the State governments could be added to the list. Hence only statement 2 is correct.

Hence, option (b) is the correct answer. Source: https://vajiramias.com/current-affairs/centrally-protected- monuments/5e5dc0101d5def6623a3bbdc/

49

Q59. Consider the following statements with reference to various schemes launched by Ministry of Culture for safeguarding culture of tribal: 1. Shilpgram aims to promote folk and tribal art in the urban areas of India. 2. Octave aims to promote and propagate the rich cultural heritage of all Himalayan states to the rest of India. Which of the statements given above is/are correct? a. 1 only b. 2 only c. Both 1 and 2 d. Neither 1 nor 2

Answer: d

Explanation:

Union Minister of Culture have launched various schemes to preserve and promote languages, folk dance, Art and culture of tribals. Some of the scheme launched by Zonal Cultural Centres (ZCCs) under Ministry of Culture are:

• Guru Shishya Parampara: This scheme envisages transmitting our valued traditions to the coming generations. Disciples are trained under veterans in art forms which are rare and vanishing. • Shilpgram: To promote folk and tribal art and crafts of the zone by organizing seminar, workshops, exhibitions, craft fairs, design development and marketing support to the artisans living in the rural areas. • Octave: To promote and propagate the rich cultural heritage of North East region comprising of eight States namely Arunachal Pradesh, Assam, Meghalaya, Mizoram, Sikkim, Nagaland, Manipur and Tripura to the rest of India. • National Cultural Exchange Programme (NCEP): Under this scheme, various festivals of performing arts, exhibitions, yatras etc are organized in member States. Hence both statements are incorrect. Hence, option (d) is the correct answer. Source: https://vajiramias.com/current-affairs/schemes-to-promote-culture-of- tribals/5e5dd3e11d5def661eccb565/ Q60. Dholavira is an archaeological site, located in:

50

a. Gujarat b. Madhya Pradesh c. Rajasthan d. Haryana Answer: a Explanation: Dholavira is an archaeological site at Khadirbet in Bhachau Taluka of , in the state of Gujarat. Dholavira’s location is on the Tropic of Cancer. Also known locally as Kotadatimba, the site contains ruins of an ancient Indus Valley Civilization/Harappan city. It is one of the five largest Harappan sites and most prominent archaeological sites in India belonging to the Indus Valley Civilization. This city was there from about 2650 BC to 1450 BC, and was first discovered in the year 1967, but excavation here began in 1989. Among prime discoveries made here were various reservoirs which bear testimony to the advanced water-management system back then. In 2014, archaeologists discovered a step-well here, which is almost three times bigger than the famous Great Bath of Mohenjo-Daro. Hence, option (a) is the correct answer. Source: https://vajiramias.com/current-affairs/dholavira/5e5dd4911d5def661eccb571/ Q61. With reference to the National Capital Territory of Delhi (NCT), consider the following statements: 1. Constitutionally the powers of law-making and administration are given to an elected legislature and the council of ministers. 2. The subject of Public order and police are directly under the jurisdiction of the Union government. Which of the statements given above is/are correct? a. 1 only b. 2 only c. Both 1 and 2 d. Neither 1 nor 2

Answer: c

Explanation:

51

The National Capital Territory of Delhi (NCT), under Article 239 AA, has been given a special status, which gives powers of law-making and administration to an elected legislature and the council of ministers. The law, however, puts two subjects — public order and police — directly under the Union government. Even here, there are exceptions. Two sections of Criminal Procedure Code (CrPC) —129 & 130 — give the Executive Magistrate certain powers relating to “unlawful assembly”. o If a group is found in unlawful assembly under Section 129 CrPC, the Executive Magistrate can issue orders to these persons to disperse. If this fails, the magistrate can use the civil force — which is the police. o If these efforts too fail, the Executive Magistrate, under Section 130 CrPC, can call an officer of the armed forces of the Union to disperse the assembly. This section states that it can be invoked for “public security”. Therefore, under these two limited powers, the Executive Magistrate, who reports to the Chief Minister, can issue orders relating to public security.The Seventh Schedule of the Constitution — which deals with the subject of public order in the state list — states that use of the armed forces in the maintenance of public order is outside the purview of the states. Hence both statements are correct. Hence, option (c) is the correct answer. Source: https://vajiramias.com/current-affairs/law-and-order- delhi/5e5f2a6b1d5def661eccf6de/ Q62. Consider the following statements: 1. All drugs are made up of two core components that is Active Pharmaceutical Ingredient (API) and excipients. 2. The Active Pharmaceutical Ingredient (API) is the part of any drug that not at all produces any effects. Which of the statements given above is/are correct? a. 1 only b. 2 only c. Both 1 and 2 d. Neither 1 nor 2

Answer: a

Explanation:

52

Statement 1 is correct: All drugs are made up of two core components: (1) Active Pharmaceutical Ingredient (API), which is the central ingredient, and (2) excipients. Statement 2 is incorrect: The Active Pharmaceutical Ingredient (API) is the part of any drug that produces its effects. Some drugs, such as combination therapies, have multiple active ingredients to treat different symptoms or act in different ways. The Government of India has recently made amendments in the export policy and restricted export of specified APIs (Active Pharmaceutical Ingredients) and formulations made from these APIs. Hence, option (a) is the correct answer. Source: https://vajiramias.com/current-affairs/active-pharmaceutical-ingredient- api/5e607d871d5def550f62b66e/ Q63. With reference to the National Interlinking of Rivers Authority (NIRA), consider the following statements: 1. It is a statutory body established by an act of Parliament in 2015. 2. As of now, only one Inter-Linking River project, the Ken-Betwa is under examination of the authorities. Which of the statements given above is/are correct? a. 1 only b. 2 only c. Both 1 and 2 d. Neither 1 nor 2

Answer: d

Explanation:

Statement 1 is incorrect: The Central government is working on the establishment of proposed National Interlinking of Rivers Authority (NIRA), an exclusive body to implement inter-State and intra-State projects. It will also make arrangements for generating funds, internally and externally.

53

Statement 2 is incorrect: As of now, six Inter-Linking River (ILR) projects — the Ken-Betwa, Damanganga-Pinjal, Par-Tapi- Narmada, Manas-Sankosh-Teesta-Ganga, -Godavari and Godavari-Cauvery (Grand Anicut) — have been under examination of the authorities. Hence, option (d) is the correct answer. Source: https://vajiramias.com/current-affairs/national-interlinking-of-rivers-authority- nira/5e607e8a1d5def551162f08a/ Q64. Which of the following kingdom used the title of Kunjali Marakkar for their naval chief? a. Kingdom of Calicut b. Bahmani Kingdom c. d. Vijayanagara Empire Answer: a Explanation: The Kunjali Marakkar or Kunhali Marakkar was the title given to the Muslim naval chief of the Zamorin of Calicut, by King of Calicut during the 16th century. There were four major Kunjalis who played a part in the Zamorin's naval wars with the Portuguese from 1507 to 1600. Kunjali Marakkars organised the first naval defense of the Indian coast by safeguarding Calicut from Portuguese invasion for almost a century. Hence, option (a) is the correct answer. Source: https://vajiramias.com/current-affairs/kunjali- marakkar/5e607f861d5def551162f0a2/ Q65. With reference to the Black carbon, consider the following statements: 1. It results from the incomplete combustion of fossil fuels and biomass. 2. It is the largest contributor to the process of climate change. Which of the statements given above is/are correct? a. 1 only b. 2 only c. Both 1 and 2

54

d. Neither 1 nor 2

Answer: a

Explanation:

Black carbon results from the incomplete combustion of fossil fuels and biomass. The fine particles absorb light and about a million times more energy than carbon dioxide. It is said to be the second largest contributor to climate change after CO2. But unlike CO2, which can stay in the atmosphere for years together, black carbon is short-lived and remains in the atmosphere only for days to weeks, before it descends as rain or snow.Hence only statement 1 is correct. Hence, option (a) is the correct answer. Source: https://vajiramias.com/current-affairs/black-carbon/5e6080c81d5def5510a919d3/ Q66. Which of the following banks have been for the Mega Consolidation? 1. Oriental Bank of Commerce and United Bank of India into Punjab National Bank 2. Syndicate Bank into Canara Bank 3. Andhra Bank and Corporation Bank into Union Bank of India 4. Allahabad Bank into Indian Bank Select the correct answer using the code given below: a. 1, 2 and 3 only b. 2 and 4 only c. 1 and 4 only d. 1, 2, 3 and 4

Answer: d

Explanation:

The Mega consolidation would help to create banks with scale comparable to global banks and capable of competing effectively in India and globally. Greater scale and synergy through consolidation would lead to cost benefits which should enable the PSBs enhance their competitiveness. Consolidation would also provide impetus to amalgamated entities by increasing their ability to support larger ticket-size lending. Further, with the adoption of technologies across the amalgamating banks, PSBs would leverage analytics in a rapidly digitalising banking landscape.

55

With this intention, the Union Cabinet has approved Mega Consolidation in Public Sector Banks {PSBs} with effect from 1.4.2020. The Union Cabinet has approved the mega consolidation of ten PSBs into four which include the – o Amalgamation of Oriental Bank of Commerce and United Bank of India into Punjab National Bank o Amalgamation of Syndicate Bank into Canara Bank o Amalgamation of Andhra Bank and Corporation Bank into Union Bank of India o Amalgamation of Allahabad Bank into Indian Bank The amalgamation results in creation of seven large PSBs with scale and national reach with each amalgamated entity having a business of over Rupees 8 lakh crore. Hence, option (d) is the correct answer. Source: https://vajiramias.com/current-affairs/consolidation-in-public-sector-banks- psbs/5e61bc781d5def550d922744/ Q67. With reference to Sodium hypochlorite, consider the following statements: 1. It is commonly used as a bleaching agent, and also to sanitise swimming pools. 2. It is not corrosive, and is meant largely to clean hard surfaces. Which of the statements given above is/are not correct? a. 1 only b. 2 only c. Both 1 and 2 d. Neither 1 nor 2

Answer: b

Explanation:

Statement 1 is correct:

Sodium hypochlorite is commonly used as a bleaching agent, and also to sanitise swimming pools. As a common bleaching agent, sodium hypochlorite is used for a variety of cleaning and disinfecting purposes. It releases chlorine, which is a disinfectant. The concentration of the chemical in the solution varies according to the purpose it is meant for. A normal household bleach (for disinfecting buildings and solid surfaces) usually is a 2-10% sodium hypochlorite solution. At a much lower 0.25-0.5%, this chemical is used to treat skin wounds like cuts or scrapes. An even weaker solution (0.05%) is sometimes used as a handwash. The World Health

56

Organization, and the US Centers for Disease Control and Prevention, recommend homemade bleach solutions of about 2-10% concentration to clean hard surfaces to clear them of any presence of the novel coronavirus. Statement 2 is incorrect:

Sodium hypochlorite is corrosive, and is meant largely to clean hard surfaces. It is not recommended to be used on human beings, certainly not as a spray or shower. Large quantities of chlorine can be harmful. A 1% solution can cause damage to the skin of anyone who comes in contact with it. If it gets inside the body, it can cause serious harm to lungs. Hence, option (b) is the correct answer. Source: https://vajiramias.com/current-affairs/sodium- hypochlorite/5e82c9ad1d5def03d84bae9a/ Q68. Consider the following statements: 1. It is a fundamental Right of every person not to be denied the right to be defended by a legal practitioner of his or her choice. 2. The Directive Principles states that equal opportunity to secure justice must not be denied to any citizen by reason of economic or other disabilities, and provides for free legal aid. Which of the statements given above is/are correct? a. 1 only b. 2 only c. Both 1 and 2 d. Neither 1 nor 2

Answer: c

Explanation:

Statement 1 is correct: Article 22(1) gives the fundamental right to every person not to be denied the right to be defended by a legal practitioner of his or her choice. Article 14 provides for equality before the law and equal protection of the laws within the territory of India. Statement 2 is incorrect:

57

Article 39A, part of the Directive Principles of state policy, states that equal opportunity to secure justice must not be denied to any citizen by reason of economic or other disabilities, and provides for free legal aid. Hence, option (b) is the correct answer. Source: https://vajiramias.com/current-affairs/right-of-an-accused-to-be- defended/5e630df21d5def6b61fb90f8/ Q69. With reference to benefits given to disabled military personnel, consider the following statements: 1. The entire pension and disability element of pension in all the categories is exempt from payment of income tax. 2. The benefits given to disabled personnel are completely independent of the percentage of disability. Which of the statements given above is/are correct? a. 1 only b. 2 only c. Both 1 and 2 d. Neither 1 nor 2

Answer: a

Explanation:

Disabled military personnel are classified under three categories: battle casualties (war wounded), battle casualties and disabilities due to service conditions. Benefits given to disabled personnel are based on the percentage of disability. The amount of disability pension is based on two categories. The first rate is for battle casualties (war wounded) and battle casualties. The second rate applies to personnel with disabilities due to service conditions.The entire pension and disability element of pension in all the categories is exempt from payment of income tax. Hence only statement 1 is correct. Hence, option (a) is the correct answer. Source: https://vajiramias.com/current-affairs/tax-on-disability- pension/5e6317c11d5def6b67b29b3b/ Q70. With reference to the VIX index, consider the following statements:

58

1. The VIX is an index that serves as a measure of market expectation of volatility in the near term. 2. The VIX index was first created by the Bombay stock Exchange and introduced in 1993. Which of the statements given above is/are correct? a. 1 only b. 2 only c. Both 1 and 2 d. Neither 1 nor 2

Answer: a

Explanation:

Statement 1 is correct: India VIX is an index that serves as a measure of market expectation of volatility in the near term. While volatility signifies the rate and magnitude of change in the stock price or index value, the movement in the VIX index reflects the overall market volatility expectations over the next 30 days. So, a spike in the VIX value means the market is expecting higher volatility in the near future.Given the nature of the index, it is also known as ‘fear gauge’ or ‘fear index’.India VIX index is not the first of its kind in the world. Statement 2 is incorrect: The VIX index was first created by the Chicago Board Options Exchange (CBOE) and introduced in 1993 based on the prices of S&P 500 index.The India VIX was launched in 2010 and is based on the computation methodology of CBOE though amended to align with the Indian markets. The CBOE and Standard & Poor's have granted a license to NSE, to use such mark in the name of the India VIX and for purposes relating to the India VIX. Hence, option (a) is the correct answer. Source: https://vajiramias.com/current-affairs/india-vix/5e82c8501d5def03d2451963/ Q71. With reference to the Himalayan Ibex, consider the following statements: 1. It is distributed in the trans-Himalayan ranges of Jammu and Kashmir, Ladakh and Himachal Pradesh up to the river . 2. It is listed as critically endangered in IUCN Red list. Which of the statements given above is/are not correct?

59

a. 1 only b. 2 only c. Both 1 and 2 d. Neither 1 nor 2

Answer: b

Explanation:

A recent study by scientists of the Zoological Survey of India (ZSI) has proved that Himalayan Ibex is a distinct species from the Siberian Ibex.Himalayan Ibex is distributed in the trans- Himalayan ranges of Jammu and Kashmir, Ladakh and Himachal Pradesh up to the river Sutlej. It is listed as Least Concernin IUCN Red list. Hence statement 1 is correct and statement 2 is incorrect. Siberian Ibex is a species of wild goat and is distributed in diverse habitats, ranging from cold deserts, rocky outcrops, steep terrain, high-land flats and mountain ridges to low mountains and foothills. From Mongolia, its distribution extends towards Altai, Hangai, Gobi-Altai, the Hurukh mountain ranges as well as Sayan Mountains near Russia and scattered populations in the small mountains of Trans-Altai Gobi. Hence, option (b) is the correct answer. Source: https://vajiramias.com/current-affairs/himalayan-ibex/5e8156dd1d5def6597300a2c/ Q72. With reference to the Commission (IOC), consider the following statements: 1. It is an intergovernmental organization that is composed of all African nations. 2. It was created in 1982 and was institutionalized in 1984 by the Victoria Agreement. Which of the statements given above is/are correct? a. 1 only b. 2 only c. Both 1 and 2 d. Neither 1 nor 2

Answer: b

60

Explanation:

India has been approved as an observer state for the Indian Ocean Commission (IOC). It is an intergovernmental organization that is composed of five African Indian Ocean nations: Comoros, Madagascar, Mauritius, Réunion (an overseas region of France), and Seychelles. It was created in 1982 and was institutionalized in 1984 by the Victoria Agreement. Its mandate is to serve as platform of solidarity for the entire population of the African Indian Ocean region. Secretariatis located in Port Louis, Mauritius. With the decision, India will join China, which was made an observer in 2016, as well as the “International Organisation of the Francophonie” or the 54-nation French-speaking collective, the European Union (EU) and Malta, which were all admitted in 2017.Hence only statement 2 is correct. Hence, option (b) is the correct answer. Source: https://vajiramias.com/current-affairs/indian-ocean- commission/5e64605e1d5def6b67b2e8b4/ Q73. The Bara-lacha Pass is a high mountain pass in which of the following mountain ranges? a. Pir Panjal b. Zanskar c. Dhauladhar d. Karakoram Answer: b Explanation: Bara-lacha la, also known as Bara-lacha Pass is a high mountain pass in Zanskar range, connecting Lahaul district in Himachal Pradesh to Leh district in Ladakh, situated along the Leh– Manali Highway. The Bhaga river, a tributary of the , originates from Surya lake, which is situated a few of kilometers from the pass towards Manali. The pass also acts as a water-divide between the Bhagariver and the Yunam river. Hence, option (b) is the correct answer. Source: https://vajiramias.com/current-affairs/bara-lacha-la/5e8150611d5def659631023f/ Q74. With reference to the Epidemic Disease Act, 1897, consider the following statements: 1. Section 2A of the Act empowers the state government to take special measures and prescribe regulations as to dangerous epidemic disease.

61

2. Section 2 of the Act empowers the central government to take steps to prevent the spread of an epidemic. Which of the statements given above is/are correct? a. 1 only b. 2 only c. Both 1 and 2 d. Neither 1 nor 2

Answer: d

Explanation:

The Epidemic Act is meant "to provide for the better prevention of the spread of dangerous epidemic diseases".Section 2 of the act empower the state government to take special measures and prescribe regulations as to dangerous epidemic disease.Section 2A of the Act empowers the central government to take steps to prevent the spread of an epidemic.According to section 3 of the act, any person disobeying any regulation or order made under this Act shall be deemed to have committed an offence punishable under section 188 of the Indian Penal Code. Hence, both statements are incorrect. Hence, option (d) is the correct answer. Source: https://vajiramias.com/current-affairs/epidemic-disease-act- 1897/5e69ab5e1d5def6b61fcf2fa/ Q75. With reference to the Additional Tier-1 bonds, consider the following statements: 1. They are a type of unsecured, perpetual bonds that banks issue to shore up their core capital base to meet the Basel-III norms.

2. They are perpetual in nature and carry call options that allow banks to redeem them after five or ten years.

Which of the statements given above is/are correct? a. 1 only b. 2 only c. Both 1 and 2 d. Neither 1 nor 2

62

Answer: c

Explanation:

AT-1, short for Additional Tier-1 bonds, are a type of unsecured, perpetual bonds that banks issue to shore up their core capital base to meet the Basel-III norms. As per RBI rules based on the Basel-III framework, AT-1 bonds have principal loss absorption features, which can cause a full write-down or conversion to equity on breach of a pre-specified trigger of common Tier 1 capital ratio falling below 6.125 per cent. AT-1 bonds have several unusual features:

• One, these bonds are perpetual and carry no maturity date. Instead, they carry call options that allow banks to redeem them after five or 10 years. • Two, banks issuing AT-1 bonds can skip interest payouts for a particular year or even reduce the bonds’ face value without getting into hot water with their investors, provided their capital ratios fall below certain threshold levels. Hence both the statements are correct. Hence, option (c) is the correct answer. Source: https://vajiramias.com/current-affairs/at-1-bonds/5e69aa9a1d5def6b61fcf2df/ Q76. With reference to malnutrition among women in India, consider the following statements: 1. More than half of the women in the age group of 15-49 years of age, are underweight. 2. Odisha has the highest percentage of malnutrition among women in India. Which of the statements given above is/are correct? a. 1 only b. 2 only c. Both 1 and 2 d. Neither 1 nor 2

Answer: d

Explanation:

The National Family Health Survey (NFHS) is a large-scale, multi-round survey conducted in a representative sample of households throughout India. The First National Family Health Survey (NFHS-1) was conducted in 1992-93.

63

As per the recent report of National Family Health Survey (NFHS) – 4 conducted by Ministry of Health and Family Welfare in 2015-16, 22.9% women (15-49 years of age) are underweight (BMI less than 18.5 kg/m2). The five States/UTs having highest percentage of malnutrition among women are Jharkhand (31.5%), Bihar (30.4%), Dadra and Nagar Haveli (28.7%), Madhya Pradesh (28.4%), Gujarat (27.2%) and Rajasthan (27%). Schemes like Anganwadi Services, Scheme for Adolescent Girls and Pradhan Mantri Matru Vandana Yojna (PMMVY) and POSHAN Aabhiyaan are being implemented to address the problem of malnutrition among women. Hence both the statements are incorrect. Hence, option (c) is the correct answer. Source: https://vajiramias.com/current-affairs/malnutrition-among- women/5e6b04f31d5def7db8bf0b25/ Q77. With reference to the Nidhi companies, consider the following statements: 1. It is a type of company in the Indian non-banking finance sector, recognized under the Companies Act, 2013. 2. They are regulated by the Ministry of Finance.

Which of the statements given above is/are correct? a. 1 only b. 2 only c. Both 1 and 2 d. Neither 1 nor 2

Answer: a

Explanation:

A Nidhi company is a type of company in the Indian non-banking finance sector, recognized under section 406 of the Companies Act, 2013. Their core business is borrowing and lending money between their members. They are regulated by Ministry of Corporate Affairs. They have to comply with two set of norms, one of Public limited company as per Companies Act, 2013 and another is for Nidhi rules, 2014. It shall have a minimum paid up equity share capital of 5,00,000. No preference shares shall be issued. Hence only statement 1 is correct. Hence, option (c) is the correct answer. Source: https://vajiramias.com/current-affairs/nidhi-company/5e6b0b531d5def2d80700911/

64

Q78. With reference to the R-naughtor R0, consider the following statements: 1. It is an epidemiologic metric used to describe the contagiousness of infectious agents. 2. If the R0 is above one, a group of infected people are less likely to spread the infection. Which of the statements given above is/are correct? a. 1 only b. 2 only c. Both 1 and 2 d. Neither 1 nor 2

Answer: a

Explanation:

The R-naught, or R0, is a virus’s basic reproductive number — an epidemiologic metric used to describe the contagiousness of infectious agents. Three possibilities exist for the potential transmission or decline of a disease, depending on its R0 value:

• If R0 is less than 1, each existing infection causes less than one new infection. In this case, the disease will decline and eventually die out. • If R0 equals 1, each existing infection causes one new infection. The disease will stay alive and stable, but there won’t be an outbreak or an epidemic. • If R0 is more than 1, each existing infection causes more than one new infection. The disease will be transmitted between people, and there may be an outbreak or epidemic. Hence only statement 1 is correct. Hence, option (a) is the correct answer. Source: https://vajiramias.com/current-affairs/r-naught/5e6b0aff1d5def6b61fd2d40/ Q79. With reference to the Essential Commodities Act 1955, consider the following statements: 1. Its objective is to regulate the production, supply and distribution of commodities declared as ‘essential’ in order to make them available to consumers at fair prices. 2. The state government can include new commodities as and when the need arises, and take them off the list once the situation improves. Which of the statements given above is/are not correct?

65

a. 1 only b. 2 only c. Both 1 and 2 d. Neither 1 nor 2

Answer: b

Explanation:

The ECA was enacted in 1955. It has since been used by the Government to regulate the production, supply and distribution of a whole host of commodities it declares ‘essential’ in order to make them available to consumers at fair prices. Additionally, the government can also fix the maximum retail price (MRP) of any packaged product that it declares an “essential commodity”. The lists of items under the Act include drugs, fertilisers, pulses and edible oils, and petroleum and petroleum products. The Centre can include new commodities as and when the need arises, and take them off the list once the situation improves. Anybody trading or dealing in the commodity, be it wholesalers, retailers or even importers are prevented from stockpiling it beyond a certain quantity. The central Government has recently declared masks (2 ply & 3 ply surgical masks, N95 masks) and hand sanitizers as Essential Commodities up to 30th June, 2020 by amending the Schedule of the Essential Commodities Act, 1955. Hence, option (b) is the correct answer. Source: https://vajiramias.com/current-affairs/masks-and-hand- sanitizers/5e6c66971d5def7db690544a/ Q80. Which of the following Ministries initiated the Bhoomi Rashi Portal? a. Ministry of Commerce and Industry b. Ministry of the Road Transport & Highways c. Ministry of Development of North Eastern Region d. Ministry of Environment, Forest and Climate Change Answer: b Explanation:

66

Bhoomi Rashi potral has significantly expedited error free and transparent land acquisition for National Highways. It was as a major e-Governance initiative of the Ministry of the Road Transport & Highways. In 2018-19, 2920 land acquisition notifications have been issued using the portal as against an average of 1000 notifications issued yearly in the previous two years. The portal has been integrated with the Public Financial Management System (PFMS) for depositing the compensation in the account of affected/ interested persons on real-time basis. The Bhoomi Rashi portal model is replicable and can be used by state governments as well as by Ministries which directly acquire land under their relevant legal provisions. Hence, option (b) is the correct answer. Source: https://vajiramias.com/current-affairs/bhoomi-rashi- potral/5e6c60701d5def7db6905308/ Q81. With reference to the National Creche Scheme, consider the following statements: 1. It is being implemented as a Central sector Scheme implemented by the Ministry of Women and Child Development.

2. It provides day care facilities to children in the age group of 6 months to 6 years of working mothers.

Which of the statements given above is/are correct? a. 1 only b. 2 only c. Both 1 and 2 d. Neither 1 nor 2

Answer: b

Explanation:

National Creche Scheme (earlier named as Rajiv Gandhi National Creche Scheme) is being implemented as a Centrally Sponsored Scheme through States/UTs being implemented by the Ministry of Women and Child Development to provide day care facilities to children (age group of 6 months to 6 years) of working mothers.

• The salient features of the National Creche Scheme are as follows: o Daycare Facilities including Sleeping Facilities. o Early Stimulation for children below 3 years and pre-school Education for 3 to 6 years old children.

67

o Supplementary Nutrition ( to be locally sourced) o Growth Monitoring and Health Check-up and Immunization Hence only statement 2 is correct. Hence, option (b) is the correct answer. Source: https://vajiramias.com/current-affairs/national-creche- scheme/5e6c5f7b1d5def7db8bf47cd/ Q82. With reference to the State Disaster Response Fund (SDRF), consider the following statements: 1. It is the primary fund available with the State Governments for responses to notified disasters. 2. The Central Government contributes 75% of the funds to the SDRF of all States/UTs. Which of the statements given above is/are correct? a. 1 only b. 2 only c. Both 1 and 2 d. Neither 1 nor 2

Answer: a

Explanation:

The State Disaster Response Fund (SDRF), constituted under Section 48 (1) (a) of the Disaster Management Act, 2005, is the primary fund available with State Governments for responses to notified disasters. The Central Government contributes 75% of SDRF allocation for general category States/UTs and 90% for special category States/UTs (NE States, Sikkim, Uttarakhand, Himachal Pradesh, Jammu and Kashmir). SDRF shall be used only for meeting the expenditure for providing immediate relief to the victims. Hence only statement 1 is correct. Hence, option (a) is the correct answer. Source: https://vajiramias.com/current-affairs/state-disaster-response-fund- sdrf/5e6db2c71d5def7db8bf8068/ Q83. With reference to the Sepsis, consider the following statements: 1. It is a life-threatening organ dysfunction caused by the body’s immune system overreacting in response to an infection.

68

2. Sepsis can be triggered by a variety of pathogens such as Viruses, bacteria, fungi or parasites. Which of the statements given above is/are correct? a. 1 only b. 2 only c. Both 1 and 2 d. Neither 1 nor 2

Answer: c

Explanation:

Statement 1 is correct: Sepsis is a life-threatening organ dysfunction caused by the body’s immune system overreacting in response to an infection. This overactive, toxic response can lead to tissue damage, multiple organ failure and death. Statement 2 is correct: Sepsis can be triggered by a variety of pathogens such as Viruses, bacteria, fungi or parasites. The causes of sepsis are usually pneumonia, wound infections, urinary tract infections or infections in the abdominal cavity. Other viruses that are highly infectious, such as coronaviruses, Ebola and yellow fever viruses, dengue, swine flu or bird flu viruses can also cause sepsis. Drop in blood pressure with a simultaneous rise in heart rate, fever, rapid, heavy breathing, an unusually strong feeling of illness and sudden confusion. Infections, kidney failure or cardiovascular problems can recur about three months after discharge. In addition, many sepsis patients suffer severe, long-term consequences such as paralysis, depression or anxiety disorders. When diagnosed, sepsis is immediately treated as an emergency. The blood is examined, a broad-spectrum antibiotic is administered, and sufficient blood circulation and ventilation are ensured. As a precautionary measure, many sepsis patients are “protectionally incubated,” i.e. put into an artificial coma. Hence, option (c) is the correct answer. Source: https://vajiramias.com/current-affairs/sepsis/5e6db13d1d5def7db8bf7f04/ Q84. With reference to the Nuclear Non-Proliferation Treaty (NPT), consider the following statements: 1. The Treaty represents the only binding multilateral commitment with a goal of nuclear disarmament.

69

2. The Treaty establishes a safeguards system under the responsibility of the International Atomic Energy Agency (IAEA). Which of the statements given above is/are not correct? a. 1 only b. 2 only c. Both 1 and 2 d. Neither 1 nor 2

Answer: d

Explanation:

Statement 1 is correct: The Nuclear Non-Proliferation Treaty (NPT) is an international treaty whose objective is to prevent the spread of nuclear weapons and weapons technology, to promote cooperation in the peaceful uses of nuclear energy and to further the goal of achieving nuclear disarmament and general and complete disarmament. The Treaty represents the only binding commitment in a multilateral treaty to the goal of disarmament by the nuclear-weapon States. Statement 2 is correct: Opened for signature in 1968, the Treaty entered into force in 1970. On 11 May 1995, the Treaty was extended indefinitely. A total of 191 States have joined the Treaty, including the five nuclear-weapon States. The Treaty establishes a safeguards system under the responsibility of the International Atomic Energy Agency (IAEA). Safeguards are used to verify compliance with the Treaty through inspections conducted by the IAEA. The treaty defines nuclear-weapon states as those that have built and tested a nuclear explosive device before 1 January 1967; these are the United States, Russia, the United Kingdom, France, and China. Four other states are known or believed to possess nuclear weapons: India, Pakistan, and North Korea have openly tested and declared that they possess nuclear weapons, while Israel is deliberately ambiguous regarding its nuclear weapons status. Hence, option (d) is the correct answer. Source: https://vajiramias.com/current-affairs/50-years-of- npt/5e6db3bb1d5def7db8bf8245/ Q85. With reference to the Sagarmala programme, consider the following statements:

70

1. Its objective is to promote port-led development in the country through harnessing India’s 7,500 km long coastline. 2. For Implementing the scheme, Government at centre and states, along with Special Purpose vehicles are playing a major role. Which of the statements given above is/are correct? a. 1 only b. 2 only c. Both 1 and 2 d. Neither 1 nor 2

Answer: c

Explanation:

The Sagarmala programme is the flagship programme of the Ministry of Shipping. Its objective is to promote port-led development in the country through harnessing India’s 7,500 km long coastline, 14,500 km of potentially navigable waterways and strategic location on key international maritime trade routes. The main vision of the Sagarmala Programme is to reduce logistics cost for EXIM and domestic trade with minimal infrastructure investment. As of now, 500 projects have been identified. Implementation of these projects are being done by the Central Line Ministries, State Governments / Maritime Boards and SPVs preferably through the private sector and through the Public Private Participation (PPP) wherever feasible. Hence both statements are correct. Hence, option (c) is the correct answer. Source: https://vajiramias.com/current-affairs/sagarmala- programme/5e703d871d5def5e61354b3a/ Q86. With reference to the Mission Solar Charkha, consider the following statements: 1. It is an initiative of Ministry of New and Renewable Energy. 2. It aims to ensure inclusive growth by generation of employment and boost rural economy and help in arresting migration from rural to urban areas. Which of the statements given above is/are correct? a. 1 only b. 2 only

71

c. Both 1 and 2 d. Neither 1 nor 2

Answer: b

Explanation:

Statement 1 is incorrect: The Ministry of Micro, Small and Medium Enterprises (MSME) has launched the Mission Solar Charkha in 2018-19 for implementation of 50 Solar Charkha Clusters across the country. Statement 2 is correct: The aims and objectives of Mission Solar Charkha are:- o To ensure inclusive growth by generation of employment, especially for women and youth and sustainable development through solar charkha cluster in rural areas. o To boost rural economy and help in arresting migration from rural to urban areas. o To leverage low-cost, innovative technologies and processes for substance. Hence, option (b) is the correct answer. Source: https://vajiramias.com/current-affairs/mission-solar- charkha/5e703e191d5def5e6562e659/ Q87. With reference to the Faster Adoption and Manufacturing of Hybrid and Electric Vehicles in India (FAME) scheme, consider the following statements: 1. It is under the administrative control of Department of Heavy Industry. 2. It was formulated to promote the manufacturing of electric and hybrid vehicle technology under the National Electric Mobility Mission Plan 2020. Which of the statements given above is/are correct? a. 1 only b. 2 only c. Both 1 and 2 d. Neither 1 nor 2

Answer: c

Explanation:

72

Faster Adoption and Manufacturing of Hybrid and Electric Vehicles in India, FAME scheme was initiated in 2015. The scheme was formulated to promote the manufacturing of electric and hybrid vehicle technology under the National Electric Mobility Mission Plan 2020. It is unbder the administrative control of Department of Heavy Industry. In the 2nd phase of the scheme, it is aimed to support through demand incentives about seven thousand E-buses, 5 lakh E-3 wheelers, 55 thousand E-4 wheelers passenger cars and 10 lakh E-2 wheelers. Hence both the statements are correct. Hence, option (c) is the correct answer. Source: https://vajiramias.com/current-affairs/electric-vehicles/5e70435f1d5def5e64e38fae/ Q88. Which of the following is the aim of Mission RakshaGyan ? a. to provide boost to the IPR culture in indigenous defence industry. b. to provide boost to Foreign Investment in defence under automatic route. c. to promote indigenous design and development of defence equipments. d. to foster innovation in Defence Sector by engaging Industries including MSMEs and start-ups. Answer: a

Explanation:

In the last two year, following steps have been taken by the defence mimisntry to boost country’s defence sector: • Industrial licensing: Defence Products list requiring Industrial Licences has been rationalised and manufacture of most of the parts and components does not require Industrial Licence. • FDI: Foreign Investment is allowed under automatic route upto 49% and above 49% through government route. • Defence Procurement Procedure (DPP): A new category of procurement ‘Buy {Indian-IDDM (Indigenously Designed, Developed and manufactured)}’ has been introduced in DPP-2016 to promote indigenous design and development of defence equipments. • “Make” Procedure: In February, 2018 a separate procedure for ‘Make-II’ sub-category has been notified wherein a number of industry friendly provisions have been introduced. • Government has notified the ‘Strategic Partnership (SP)’ Model which envisages establishment of long-term strategic partnerships with Indian entities, wherein they would tie up with global Original Equipment Manufacturers (OEMs) to seek technology transfers to set up domestic manufacturing infrastructure and supply chains. • iDEX: Innovations for Defence Excellence (iDEX) framework, was launched with the aim to achieve self-reliance and to foster innovation in Defence Sector by engaging Industries including MSMEs, startups, individual innovators, R&D institutes and academia.

73

• Indigenisation policy: Government has notified a Policy for indigenisation of components and spares used in Defence Platforms in March, 2019 with the objective to indigenize the imported components (including alloys & special materials) and sub-assemblies for defence equipment and platforms manufactured in India. • Defence Corridors: Government has decided to establish two defence industrial corridors in Tamil Nadu and Uttar Pradesh to serve as engines of economic development and growth of defence industrial base in the country. • The Ministry has instituted a new framework titled ‘Mission RakshaGyan Shakti’ which aims to provide boost to the IPR culture in indigenous defence industry. • Defence Investor Cell has been created in the Ministry to provide all necessary information including addressing queries related to investment opportunities, procedures and regulatory requirements for investment in the sector. Hence, option (a) is the correct answer. Source: https://vajiramias.com/current-affairs/fdi-in-defence- sector/5e7984861d5def186274b298/ Q89. With reference to the Biological Weapons Convention, consider the following statements: 1. The Biological Weapons Convention (BWC) is a legally binding treaty that outlaws biological arms. 2. It is the first multilateral disarmament treaty to ban an entire category of weapons of mass destruction. Which of the statements given above is/are correct? a. 1 only b. 2 only c. Both 1 and 2 d. Neither 1 nor 2

Answer: c

Explanation:

Statement 1 is correct: The Convention on the Prohibition of the Development, Production and Stockpiling of Bacteriological (Biological) and Toxin Weapons and on their Destruction is usually referred to as the Biological Weapons Convention (BWC) or Biological and Toxin Weapons Convention

74

(BTWC). The Biological Weapons Convention (BWC) is a legally binding treaty that outlaws biological arms. March 26 marked the 45th anniversary of the entry into force of the Biological Weapons Convention. Statement 2 is correct: The BWC is the first multilateral disarmament treaty to ban an entire category of weapons of mass destruction. The BWC opened for signature in 1972, and entered into force in 1975. It currently has 183 states-parties, including Palestine, and four signatories. Ten states have neither signed nor ratified the BWC. The development, stockpiling, acquisition, retention, and production of: Biological agents and toxins "of types and in quantities that have no justification for prophylactic, protective or other peaceful purposes;" Weapons, equipment, and delivery vehicles "designed to use such agents or toxins for hostile purposes or in armed conflict." The transfer of or assistance with acquiring the agents, toxins, weapons, equipment, and delivery vehicles described above. The BWC does not ban the use of biological and toxin weapons but reaffirms the 1925 Geneva Protocol, which prohibits such use. It also does not ban biodefense programs. Hence both the statements are correct. Hence, option (c) is the correct answer. Source: https://vajiramias.com/current-affairs/biological-weapons- convention/5e81533b1d5def65973009ec/ Q90. Consider the following statements with reference to Section 144 of the Criminal Procedure Code (CrPC) of 1973: 1. It authorizes the Executive Magistrate of any state or territory to issue an order to prohibit the assembly of four or more people in an area. 2. Its implementation warrants much graver situation posing bigger danger of rioting and violence than curfew. Which of the statements given above is/are correct? a. 1 only b. 2 only c. Both 1 and 2 d. Neither 1 nor 2

Answer: a

Explanation:

75

Section 144 of the Criminal Procedure Code (CrPC) of 1973 authorises the Executive Magistrate of any state or territory to issue an order to prohibit the assembly of four or more people in an area. Restriction under Section 144 is different from curfew. In the areas where curfew is imposed, all public activity is barred. Civilian traffic is also stopped. Curfew warrants much graver situation posing bigger danger of rioting and violence.Hence only statement 1 is correct. Hence, option (a) is the correct answer. Source: https://vajiramias.com/current-affairs/janta-curfew/5e7447981d5def5e64e44577/ Q91. Mahatma Gandhi Bunkar Bima Yojana (MGBBY) is an initiative of: a. Ministry of Chemicals and Fertilizers b. Ministry of Environment, Forest and Climate Change c. Ministry of Petroleum and Natural Gas d. Ministry of Textiles Answer: d Explanation: The Ministry of Textiles is implementing converged Mahatma Gandhi Bunkar Bima Yojana (MGBBY) for providing social security benefits like life, accidental & disability insurance coverage to handloom weavers/workers in the age group of 51-59 years across the country, who have already enrolled under the scheme. The claim benefits are provided by LIC directly into the bank account of beneficiaries through Direct Benefit Transfer (DBT). Hence, option (d) is the correct answer. Source: https://vajiramias.com/current-affairs/mahatma-gandhi-bunkar-bima-yojana- mgbby/5e7591701d5def5e64e478e2/ Q92. With reference to the Pardoning Power of President of India, consider the following statements: 1. Article 92 of the Constitution states that he/she can grant pardons, reprieves, respites or remissions of punishment or to suspend, remit or commute the convict. 2. The mercy petition is reviewed by the Ministry of Home Affairs, which consults the state involved, before going to the President. Which of the statements given above is/are correct? a. 1 only b. 2 only

76

c. Both 1 and 2 d. Neither 1 nor 2

Answer: b

Explanation:

According to Article 72 of the Constitution, the power to pardon — philosophy of which is “every civilised country recognises and provides for the pardoning power as an act of grace and humanity in course of law” — lies with the President.The Article also states that he/she can grant pardons, reprieves, respites or remissions of punishment or to suspend, remit or commute the convict.The mercy petition is reviewed by the Ministry of Home Affairs, which consults the state involved, before going to the President. According to the Criminal Procedure Code, hanging is the method of execution in the civilian court system. The Army Act, 1950, however, lists both hanging and shooting as official methods of execution in the military court-martial system. The executions were carried out after the convicts exhausted every possible legal avenue to escape the gallows. Hence only statement 2 is correct. Hence, option (b) is the correct answer. Source: https://vajiramias.com/current-affairs/capital- punishment/5e7593bd1d5def5e64e47969/ Q93. The Réunion Island is located in which of the following regions? a. Indian Ocean b. Pacific Ocean c. Atlantic Ocean d. Arctic Ocean Answer: a Explanation: Réunion is an overseas department and region of the France and an island in the Indian Ocean, east of Madagascar and southwest of Mauritius. It is above a hotspot in the Earth's crust. The Piton de la Fournaise, a shield volcano on the eastern end of Réunion Island, rises more than 2,631 m above sea level. India and France have recently conducted joint patrols from the Reunion Island for the first time. Hence, option (a) is the correct answer.

77

Source: https://vajiramias.com/current-affairs/india-france-defence- relations/5e76e0b51d5def05d26990b2/ Q94. Project Swastik is an initiative of: a. Border Roads Organisation b. Central Reserve Police Force c. Indo Tibetan Border Police d. Sashastra Seema Bal Answer: a Explanation: Recently, 360 feet long bailey suspension bridge has been opened for traffic over Teesta River in Munshithang near Chungthang town. This Bridge was constructed by Border Roads Organisation (BRO) over Teesta River in North Sikkim under Project Swastik. The bridge will give impetus to tourism and facilitate the movement of logistics for the Armed Forces deployed in forward areas. Hence, option (a) is the correct answer. Value Addition: Project Arunank is responsible for the construction and maintenance of approximately 1113 Kms of roads in Arunachal Pradesh and Assam. It is being implemented by the Border Roads Organisation, an important arm of the Ministry of Defence. Project Arunank is named after the state of Arunachal Pradesh. Hence, option (a) is the correct answer. Source: https://vajiramias.com/current-affairs/project-swastik/5e76ddb61d5def05d74372f2/ Q95. With reference to the Modified Electronics Manufacturing Clusters (EMC 2.0) Scheme, consider the following statements: 1. It would support setting up of both Electronics Manufacturing Clusters (EMCs) and Common Facility Centers (CFCs). 2. It is an initiative of all the major private electronic manufacturing companies in India with NITI Aayog. Which of the statements given above is/are not correct? a. 1 only b. 2 only

78

c. Both 1 and 2 d. Neither 1 nor 2

Answer: b

Explanation:

The Modified Electronics Manufacturing Clusters (EMC 2.0) Scheme of Ministry of Electronics and Information Technology (MeitY) would support setting up of both Electronics Manufacturing Clusters (EMCs) and Common Facility Centers (CFCs). An Electronics Manufacturing Cluster (EMC) would set up in geographical areas of certain minimum extent, preferably contiguous, where the focus is on development of basic infrastructure, amenities and other common facilities for the ESDM units. For Common Facility Centre (CFC), there should be a significant number of existing ESDM units located in the area and the focus is on upgrading common technical infrastructure and providing common facilities for the ESDM units in such EMCs, Industrial Areas/Parks/industrial corridors. Hence statement 1 is correct and statement 2 is incorrect. Hence, option (b) is the correct answer. Source: https://vajiramias.com/current-affairs/modified-electronics-manufacturing-clusters- emc-20-scheme/5e76df351d5def05d55e3321/ Q96. Which of the following sectors have been granted 100% Foreign direct investment (FDI) under the Automatic Route? 1. Defence 2. White Label ATM Operations 3. Asset Reconstruction Companies 4. Tourism & Hospitality Select the correct answer using the code given below: a. 1 and 3 only b. 2, 3 and 4 only c. 2 and 4 only d. 1, 3 and 4 only

Answer: b

79

Explanation:

In 2001, the Defence Industry sector, which was hitherto reserved for the public sector, was opened upto 100% for Indian private sector participation, with FDI upto 26% both subject to licensing. Further, Department for Promotion of Industry and Internal Trade, Ministry of Commerce & Industry has allowed FDI under automatic route upto 49% and above 49% through government route wherever it is likely to result in access to modern technology. Further, FDI in defence industry sector is subject to industrial license under Industries (Development & Regulation) Act, 1951 and manufacturing of small arms and ammunition under the Arms Act, 1959. Sectors which come under the ' 100% Automatic Route' category are Agriculture & Animal Husbandry, Air-Transport Services (non-scheduled and other services under civil aviation sector), Airports (Greenfield + Brownfield), Asset Reconstruction Companies, Auto-components, Automobiles, Biotechnology (Greenfield), Broadcast Content Services (Up-linking & down-linking of TV channels, Broadcasting Carriage Services, Capital Goods, Cash & Carry Wholesale Trading (including sourcing from MSEs), Chemicals, Coal & Lignite, Construction Development, Construction of Hospitals, Credit Information Companies, Duty Free Shops, E-commerce Activities, Electronic Systems, Food Processing, Gems &Jewellery, Healthcare, Industrial Parks, IT & BPM, Leather, Manufacturing, Mining & Exploration of metals & non-metal ores, Other Financial Services, Services under Civil Aviation Services such as Maintenance & Repair Organizations, Petroleum & Natural gas, Pharmaceuticals, Plantation sector, Ports & Shipping, Railway Infrastructure, Renewable Energy, Roads & Highways, Single Brand Retail Trading, Textiles & Garments, Thermal Power, Tourism & Hospitality and White Label ATM Operations. Hence, option (b) is the correct answer.

Source: https://vajiramias.com/current-affairs/fdi-in-defence- sector/5e7984861d5def186274b298/ Q97. With reference to the National Supercomputing Mission, consider the following statements: 1. It is steered jointly by the Ministry of Electronics and IT (MeitY) and Department of Science and Technology (DST). 2. The target of the mission is to establish a network of supercomputers in all academic and research institutions across the country by 2030. Which of the statements given above is/are correct? a. 1 only

80

b. 2 only c. Both 1 and 2 d. Neither 1 nor 2

Answer: a

Explanation:

India’s National Supercomputing Mission (NSM) was set up to meet the increasing computational demands of academia, researchers, MSMEs, and startups by creating the capability design, manufacturing, of supercomputers indigenously in India.National Super Computing Mission is steered jointly by the Ministry of Electronics and IT (MeitY) and Department of Science and Technology (DST) and implemented by the Centre for Development of Advanced Computing (C-DAC), Pune and the Indian Institute of Science (IISc), Bengaluru. The target of the mission was set to establish a network of supercomputers ranging from a few Tera Flops (TF) to Hundreds of Tera Flops (TF) and three systems with greater than or equal to 3 Peta Flops (PF) in academic and research institutions of National importance across the country by 2022. The first supercomputer assembled indigenously, called ParamShivay, was installed in IIT (BHU) and was inaugurated by the Prime Minister. Similar systems Param Shakti and Param Brahma were installed at IIT-Kharagpur and IISER, Pune. Hence only Statement 1 is correct. Hence, option (a) is the correct answer.

Source: https://vajiramias.com/current-affairs/national-supercomputing- mission/5e7982d41d5def1867768e94/ Q98. Which of the following festivals celebrated across India, mark the onset of New Year? 1. Yugadi 2. Gudi Padwa 3. Cheti Chand Select the correct answer using the code given below: a. 1 only b. 1 and 2 only c. 2 and 3 only d. 1, 2 and 3 81

Answer: d

Explanation:

The following festival celebrated across India marks the beginning of New Year:

• Yugadi (also known as Samvatsaradi or ): It is the New Year's Day for the people of Karnataka, Andhra Pradesh and Telangana states in India. It is observed in these regions on the first day of month of Chaitra (the first month of lunar calendar). • Gudi Padwa: Maharashtrians term celebrate the festival as GudiPadwa. • Cheti Chand: Sindhis celebrate the same day as Cheti Chand. • Sajibu Nongma Panba (also called Meetei Cheiraoba or Sajibu Cheiraoba): Manipuris also celebrate the same day as SajibuNongmaPanba. • Traditionally, these festivals mark the onset of New Year and the warmer months and they reflect values of prosperity and well-being. Hence, option (d) is the correct answer. Source: https://vajiramias.com/current-affairs/ugadi/5e7ad89c1d5def186274ea87/ Q99. Consider the following pairs:

Which of the pairs given above is/are correct? a. 1 and 2 only b. 2 and 3 only c. 1 and 3 only d. 1, 2 and 3 Answer: c

Explanation:

Pair 1 is correct: National Centre for Disease Control (NCDC) is under administrative control of the Director General of Health Services, Ministry of Health and Family Welfare, Govt. of India. The Director, 82

an officer of the Public Health sub-cadre of Central Health Service, is the administrative and technical head of the Institute. The National Centre for Disease Control (NCDC), formerly National Institute of Communicable Diseases (NICD) had its origin as Central Malaria Bureau, established at Kasauli (Himachal Pradesh) in 1909. It functions as the nodal agency in the country for disease surveillance facilitating prevention and control of communicable diseases. Headquarters is at New Delhi. Pair 2 is incorrect: The Central Drugs Standard Control Organisation(CDSCO) under Directorate General of Health Services, Union Ministry of Health is the National Regulatory Authority (NRA) of India. Its headquarters in New Delhi.Under the Drugs & Cosmetics Act,1940, CDSCO is responsible for approval of Drugs, Conduct of Clinical Trials, laying down the standards for Drugs, control over the quality of imported Drugs in the country. Pair 3 is correct: Petroleum & Explosives Safety Organization (PESO) is an organization under Department of industrial policy and promotion (DIPP), Ministry of commerce & industry. It administers the usage of explosives & petrol stations in India. It is headquartered at Nagpur, Maharashtra. Hence, option (c) is the correct answer. Source: https://vajiramias.com/current-affairs/national-centre-for-disease-control- ncdc/5e7ad9991d5def186834f180/ Q100. With reference to the Members of Parliament Local Area Development Scheme (MPLADS), consider the following statements: 1. The Scheme is under the supervision of the Ministry of Rural Development. 2. The funds released under the Scheme are lapsable i.e., the entitlement of funds released in a particular year is not carried forward to the subsequent years. Which of the statements given above is/are correct? a. 1 only b. 2 only c. Both 1 and 2 d. Neither 1 nor 2

Answer: d

Explanation:

83

• The Members of Parliament Local Area Development Scheme (MPLADS) was launched in 1993. • Initially, Ministry of Rural Development was the Nodal Ministry for this scheme. In October, 1994 this scheme was transferred to the Ministry of Statistics and Programme Implementation. • The Scheme is fully funded by the Government of India under which funds are released in the form of grants-in-aid directly to the district authorities. • The funds released under the Scheme are non-lapsable, i.e., the entitlement of funds not released in a particular year is carried forward to the subsequent years, subject to eligibility. • At present, the annual entitlement per MP/ constituency is ₹ 5 crore. • Under it, the role of the Members of Parliament is limited to recommend works. Thereafter, it is the responsibility of the district authority to sanction, execute and complete the works recommended within the stipulated time period. o The elected LokSabha Members can recommend works in their respective constituencies. o The elected members of the RajyaSabha can recommend works anywhere in the state from which they are elected. o Nominated Members of the Parliament can recommend works for implementation, anywhere in the country. • MPLADS works can be implemented in areas affected by natural calamities like floods, cyclone, hailstorm, avalanche, cloudburst, pest attack, landslides, tornado, earthquake, drought, tsunami, fire and biological, chemical, radiological hazards, etc. • In order to accord special attention to the development of areas inhabited by Scheduled Castes (SCs) and Scheduled Tribes (STs), 15 per cent of MPLADS funds are to be utilized for areas inhabited by SC population and 7.5 per cent for areas inhabited by ST population. Hence both statements are incorrect. Source: https://vajiramias.com/current-affairs/members-of-parliament-local-area- development-scheme-mplads/5e7adaea1d5def186776bf68/

84